Peds Flashcards

1
Q

Craniosynostosis

A

“Craniosynostosis” is a fancy word for premature fusion o f one or several o f the cranial
sutures. The consequence o f this premature fusion is a weird looking head and face (with
resulting difficulty getting a date to the prom). Besides looking like a gremlin (or a coneheaded
extraterrestrial forced to live as a typical suburban human), these kids can also have
increased intracranial pressure, visual impairment, and deafness.
There are different named types depending on the suture involved - thus it’s worth spending
a moment reviewing the names and locations of the normal sutures.

How well did you know this?
1
Not at all
2
3
4
5
Perfectly
2
Q

Metopic suture

A

Bones Involved: frontal

Fusion direction: front to back

Fusion order: first ( 2-3months)

How well did you know this?
1
Not at all
2
3
4
5
Perfectly
3
Q

coronal suture

A

Bones Involved: frontal and parietal

Fusion direction: lateral to medial

Fusion order: second

How well did you know this?
1
Not at all
2
3
4
5
Perfectly
4
Q

lambdoid suture

A

Bones Involved: parietal and occipital

Fusion direction: lateral to medial

Fusion order: third

How well did you know this?
1
Not at all
2
3
4
5
Perfectly
5
Q

sagitaal suture

A

Bones Involved: parietal

Fusion direction: back to front

Fusion order: fourth

How well did you know this?
1
Not at all
2
3
4
5
Perfectly
6
Q

Sutures overview

A

• Sutures normally have a serrated (saw tooth) contour
• With early closure the suture will lose the serrated appearance - becoming more dense
and sharp. Eventually the suture will disappear completely.
• For the purpose of multiple choice, you should think about synostosis (early closure) as
likely syndromic - and focus your memorizing on this point.
• Having said that, sagittal and unilateral coronal synostosis are typically idiopathic

How well did you know this?
1
Not at all
2
3
4
5
Perfectly
7
Q

Metopic Synostosis

A

trigonocephaly

Eyes are close together (hypotelorism)
Ethmoid sinuses underdeveloped
Medial part of the orbit slants up
Single suture synostosis most frequently
associated with cognitive disorders (growth
restriction of the frontal lobes)

“Quizzical Eye” appearance

How well did you know this?
1
Not at all
2
3
4
5
Perfectly
8
Q

sagittal syostosis

A

scaphocephaly or Dolichocephalic

Looks like an upside-down boat.
Usually the kids have a normal IQ
Usually the kids do NOT have hydrocephalus
Associated with Marfans
(both are tall and skinny).

most common form

How well did you know this?
1
Not at all
2
3
4
5
Perfectly
9
Q

coronal synostosis

A

brachycephaly

Unilateral subtype is more common.
Unilateral type causes the ipsilateral orbit to
elevate, and contralateral frontal bone to
protrude “frontal bossing”
Bilateral form is Rare - should make you think
syndromes (Borat s brother Bilo *).

“HarlequinEye” * if unilateral.

How well did you know this?
1
Not at all
2
3
4
5
Perfectly
10
Q

lambdoid synostosis

A

turricephaly

Tall Cranium (oxycephaly, acrocephaly)
See Next Page for Unilateral Discussion

Least
Common
Form

How well did you know this?
1
Not at all
2
3
4
5
Perfectly
11
Q

Plagiocephaly

A
This word basically means “flat.”
You will see it used to describe
unilateral coronal synostosis as
“anterior plagiocephaly
You will see it used to describe
unilateral lambdoid synostosis as
“posterior plagiocephaly. ”
The problem is that many people
use the word “plagiocephaly” to
describe the specific entity of
“deformationalplagiocephaly ’’ -
which is just benign positional
molding, not a pathologic early
closure. On the following page, I’ll
go into more detail on this. Just
know you may be required to read
the question writer’s mind when the
word is used to differentiate
between the benign and pathologic
entities.
How well did you know this?
1
Not at all
2
3
4
5
Perfectly
12
Q

POSITIONAL

PLAGIOCEPHALY

A
Infants that sleep on the same side every night
develop a flat spot on the preferred dependent
area of the head (occipital flattening).
Onset: Weeks After Birth
Ipsilateral Ear: Anterior
Frontal Bossing: Ipsilateral
Most common cause of an abnormal
skull shape in infant
Management is conservative
(sleep on the other side for a bit)
How well did you know this?
1
Not at all
2
3
4
5
Perfectly
13
Q

LAMBDOID

CRANIOSYNOSTOSIS

A
I f this is bilateral think underlying
Rhombencephalosynapsis
Onset: Birth
Ipsilateral Ear: Posterior / Inferior
Frontal Bossing: Contralateral
Rare as Fuck
Management is Surgery
How well did you know this?
1
Not at all
2
3
4
5
Perfectly
14
Q

THIS vs THAT: Positional Plagiocephaly vs True Unilateral Lambdoid Synostosis

next step

A

Outside of the jungle (or the year 1987), the
diagnosis of synostosis is going to be made with
CT + 3D. If asked what test to order I would say
CT with 3D recons. Having said that, they could
show you a skull plain film (from 1987) and ask
you to make the diagnosis on that.
If the test writer was feeling particularly cruel
and bitter he/she could show the diagnosis
with ultrasound. In that case, remember that
a normal open suture will appear as an
uninterrupted hypoechoic fibrous gap
between hyperechoic cranial bones (Bright -
Dark - Bright, Bone - Suture - Bone).
Although certain MR gradient sequences can
be used, MRI has traditionally been
considered unreliable in identifying sutures
individually.

How well did you know this?
1
Not at all
2
3
4
5
Perfectly
15
Q

THIS vs THAT: Positional Plagiocephaly vs True Unilateral Lambdoid Synostosis

trivia

A
^or PurPose muP’ple
choice, there are numerous
random bone buzzwords
that are supposed to elicit
the reflexive diagnosis
NF-l in your brain when
you hear / read them.
The more common ones include:
• Absence / Dysplasia of the Greater
Sphenoid Wing,
• Tibial Pseudoarthrosis,
• Scoliosis, and
• Lateral Thoracic Meningocele.
I’d like to add "bone defect in the region
o f the lambdoid suture ” or the “asterion
defect ” to that list of reflex generators.
It’s rare and poorly described - therefore
potentially high yield.
How well did you know this?
1
Not at all
2
3
4
5
Perfectly
16
Q

Clover Leaf Skull Syndrome

A

• Also referred to as Kleeblattschadei for the purpose of
fucking with you
• Contrary to what the name might imply - this complex
deformity is not associated with an increased ability to hit
green lights, reliably find good parking spots, or win the
lottery. I think that’s because the shape is more 3 leaf clover,
and not 4 leaf. One might assume, a head shaped like a 4 leaf clover
would probably be luckier.
• Instead, this deformity is characterized by enlargement o f the head
with a trilobed configuration, resembling a three-leaved clover.
• Results from premature synostosis o f coronal and lambdoid sutures
(most commonly), but often the sagittal closes as well.
• Hydrocephalus is a common finding.
• Syndromic Associations: Thanatophoric dysplasia, Apert syndrome
(severe), Crouzon syndrome (severe)

How well did you know this?
1
Not at all
2
3
4
5
Perfectly
17
Q

Clover Leaf Skull Syndrome

what sutures are closed

A

All the sutures are
closed
except the metric
and squamosal

How well did you know this?
1
Not at all
2
3
4
5
Perfectly
18
Q

Additional Craniosynostosis Syndromes

A

Most of the time (85%) premature closure is a primary (isolated) event, although
it can occur as the result o f a syndrome (15%). The two syndromes worth having
vague familiarity with are Apert’s and Crouzon’s

How well did you know this?
1
Not at all
2
3
4
5
Perfectly
19
Q

Apert’s

A
  • Brachycephaly (usually)

• Fused Fingers (syndactyly) - “sock hand’

How well did you know this?
1
Not at all
2
3
4
5
Perfectly
20
Q

Crouzon’s

A
  • Brachycephaly (usually)
  • 1 st Arch structures (maxilla and mandible hypoplasia).
  • Associated with patent ductus arteriosus and aortic coarctation.
  • Short central long bones (humerus, femur) - “rhizomelia”
  • Chiari 1 malformations 3: -70% of cases
How well did you know this?
1
Not at all
2
3
4
5
Perfectly
21
Q

Convolutional Markings

A
Normal gyral impressions on
the inner table of the skull.
You see them primarily during
normal rapid brain growth (age
3-7).
Usually mild and favors the
posterior skull.
If you see them along the more
anterior skull then you should
think about a “copper beaten”
skull from the increased
intracranial pressure.
How well did you know this?
1
Not at all
2
3
4
5
Perfectly
22
Q

Copper Beaten

A
The same thing as convolutional
markings (the normal gyral
impressions), just a shit ton
more of them.
You also see them along the
anterior portions of the skull not
just the posterior.
Think about things that cause
increased intracranial
pressure.
Classic examples:
• Craniosynostosis
• Obstructive Hydrocephalus
How well did you know this?
1
Not at all
2
3
4
5
Perfectly
23
Q

Luckenschadel - “L a c u n a r”

A
Oval, round, and finger shaped
defects (craters) within the
inner surface of the skull
Different than Copper Beaten
in that:
(A) They aren’t gyrifonn.
(B) They aren’t related to
increased ICP.
(C) They are usually present at
birth.
Instead they are the result of
defective bone matrix.
Classic Association:
• Chiari II malformation /
Neural Tube Defects.
How well did you know this?
1
Not at all
2
3
4
5
Perfectly
24
Q

lytic Skull Lesions

A

Lytic skull lesions in kids can come from a couple o f different things (LCH, Infection, Mets,
Epidermoid Cysts, Leptomeningeal Cysts, etc…). The two 1 want you to focus on are LCH and
the Leptomeningeal Cyst (which I will discuss later in the chapter).

How well did you know this?
1
Not at all
2
3
4
5
Perfectly
25
Q

LCH

A

(Langerhans Cell Histiocytosis) - Too many fucking dendritic cells - with local invasion. It
is a sorta pseudo malignancy thing. Nobody really understands it…. For the purpose of the exam
think about this as a beveled hole in the skull. The skull is the most common bone involved with
LCH. It is a pure lytic lesion (no sclerotic border). The beveled look is because it favors the
inner table. It can also produce a sequestrum o f intact bone (“button sequestrum).

How well did you know this?
1
Not at all
2
3
4
5
Perfectly
26
Q

lytic skull lesions

gamesmanship

A

If they tell you (or infer) the kid has neuroblastoma

- think about a met.

How well did you know this?
1
Not at all
2
3
4
5
Perfectly
27
Q

Parietal Foramina

A

These paired, mostly round, defects in the
parietal bones represent benign congenital
defects. The underlying cause is a delayed or
incomplete ossification in the underlying
parietal bones.
They can get big and confluent across the
midline. Supposedly, (at least for the big ones -
> 5mm) they are associations with cortical and
venous anomalies.

How well did you know this?
1
Not at all
2
3
4
5
Perfectly
28
Q

Wormian Bones

A

In technical terms, there are a bunch o f extra squiggles around the lambdoid sutures.
“Intrasutural Bones” they call them.
These things are usually idiopathic - however, if
you see more than 10 you should start thinking
syndromes.

How well did you know this?
1
Not at all
2
3
4
5
Perfectly
29
Q

Wormian Bones

gamesmanship

A

< 10 = Idiopathic
> 10 = First think Osteogenesis Imperfecta
> 10 + Absent Clavicle = Cleidocranial Dysostosis

How well did you know this?
1
Not at all
2
3
4
5
Perfectly
30
Q

Wormian Bones

Ddx

A

There is a massive differential, but 1 would just remember these “PORK-CHOP”

Pyknodysostosis
Osteogenesis Imperfecta
Rickets
Kinky Hair Syndrome
(Menke s / Fucked Copper Metabolism)
Cleidocranial Dysostosis
Hypothyroidism / Hypophosphatasia
One too many 21 st chromosomes (Downs)
Primary Acro-osteolysis (Hajdu-Cheney)
How well did you know this?
1
Not at all
2
3
4
5
Perfectly
31
Q

Dermoid / Epidermoid of the Skull

A
In the context of the skull, you can
think about these things as occurring
from the congenital misplacement of
cells from the scalp into the bony
calvarium.
The result is a growing lump o f tissue
(keratin debris, skin glands, etc...)
creating a bone defect with benign
appearing sclerotic borders.
There are a few differences between
the two subtypes that could be
potentially testable (contrasted
masterfully in the chart).
Although, I suspect a “what is it ? ”
type question is more likely. As such,
look through some google image
examples to prepare yourself for that
contingency.
How well did you know this?
1
Not at all
2
3
4
5
Perfectly
32
Q

Epidermoid

A

Histology:Only Skin
(Squamous
Epithelium)

Age of onset:Present between
age 20-40

Location: Present between
age 20-40

CT: CSF Density

MRI:T1 Variable,
T2 Bright,
NO Enhancement

How well did you know this?
1
Not at all
2
3
4
5
Perfectly
33
Q

Dermoid

A

Histology: Skin + Other Stuff
Like Hair Follicles,
Sweat Glands Etc

Age of onset:Typically have an
earlier presentation

Location: end to be midline.
The skin ones tend to
he around the orbits.

Associated with
Encephaloceles -
especially when
midline

CT: - More Heterogeneous,
- Calcifications
(internal or peripheral)
may be present

MRI: T2 Bright,
+/- Wall Enhancement

How well did you know this?
1
Not at all
2
3
4
5
Perfectly
34
Q

Congenital Dermal Sinus

A

Usually when people talk about these things they are referring to the spina bifida style
midline lumbosacral region defects. However, we are going to stay focused on the skull /
face. The two classic locations for dermal communications with the dura are the occiput and
the nose. Both of which are classically midline, and can be associated with a dermoid cyst.
For gamesmanship, consider a sinus tract anytime you see a cyst in these locations.

Dural communication will require
communication through the foramen cecum.

Sinus tracts may or
may not have
associated dennal
or intraosseous
cysts.
Cysts may or may
not have sinus
tracts
How well did you know this?
1
Not at all
2
3
4
5
Perfectly
35
Q

THIS vs THAT: Scalp Trauma

A

There are 3 scalp hematoma subtypes. Because the subtypes are fairly similar, there is a high
likelihood a sadistic multiple choice writer will attempt to confuse you on the subtle
differences

How well did you know this?
1
Not at all
2
3
4
5
Perfectly
36
Q

Subgaleal Hemorrhage

A

Location:Deep to the Aponeurosis
(between aponeurosis and
periosteum)

Suture:NOT limited by
suture lines

Trivia: Covers a much larger area
than a cephalohematoma

Complications:Potentially life-threatening
- rapid blood loss.
Often not seen until 12-72
hours post delivery.

Cause: vacuum extraction

How well did you know this?
1
Not at all
2
3
4
5
Perfectly
37
Q

Cephalohematoma

A

Location: Under the Periosteum
(skin o f the bone)

Suture: Limited by suture lines
(won i cross sutures)

Trivia:Outer border may calcify as a rim
and leave a deformity - sorta like
a myositis ossificans

Complications:Usually requires no intervention
(resolves within a few weeks)
Can get super infected (E.Coli).
Abscess would require drainage.
Can cause skull osteomyelitis

Cause:Instrument or Vacuum Extraction

How well did you know this?
1
Not at all
2
3
4
5
Perfectly
38
Q

Caput Succedaneum

A

Location:Subcutaneous
Hemorrhage
(superficial to
the aponeurosis)

Suture:NOT limited by
suture lines

Trivia:Requires no
intervention
(resolves within
a few days)

Cause: prolonged delivery

How well did you know this?
1
Not at all
2
3
4
5
Perfectly
39
Q

Skull Fractures

A

Accidental (and non-accidental) head trauma is supposedly (allegedly, allegedly) the most common
cause of morbidity and mortality in children. As you might imagine, the pediatric skull can fracture
just like the adult skull - with linear and comminuted patterns. For the purpose of multiple choice, I
think we should focus on the fracture patterns that are more unique to the pediatric population:
Diastatic, Depressed, and “Ping-Pong”

How well did you know this?
1
Not at all
2
3
4
5
Perfectly
40
Q

Diastatic Fracture

A

This is a fracture along / involving the suture. When they intersect it is usually
fairly obvious. It can get tricky when the fracture is confined to the suture itself. The most common
victim of this sneaky fracture is usually the Lambdoid, followed by the Resident reading the case on
night float .. .with Attending backup (asleep in bed). How does one know there is traumatic injury to
a suture ? Classically, it will widen. This is most likely to be shown in the axial or coronal plane so
you can appreciate the asymmetry ( > 1 mm asymmetry relative to the other side).

How well did you know this?
1
Not at all
2
3
4
5
Perfectly
41
Q

Depressed Fracture

A

This is a fracture with inward displacement of the bone. How much inward
displacement do you need to call it “depressed” ? Most people will say “equal or greater to the
thickness of the skull.” Some people will use the word “compound” to describe a depressed
fracture that also has an associated scalp laceration. Those same people may (or may not) add the
word “penetrating” to describe a compound fracture with an associated dural tear.
Will any o f those people be writing the questions ? The dark side clouds everything. Impossible to
see the future is.

How well did you know this?
1
Not at all
2
3
4
5
Perfectly
42
Q

Ping Pong Fracture

outcomes

A

Ping Pong fractures typically have a favorable / benign clinical outcome
(depressed fractures have high morbidity).

How well did you know this?
1
Not at all
2
3
4
5
Perfectly
43
Q

Ping Pong Fracture

etiology

A

Diastatic and depressed fracture types usually require a significant wack on
the head. Where as “ping pong” fractures often occur in the setting of birth trauma
(Mom’s pelvic bones +/- forceps).

How well did you know this?
1
Not at all
2
3
4
5
Perfectly
44
Q

Ping Pong Fracture

imaging appearnce

A

Ping Pong fractures are hard as fuck to see. To show this on a
test you’d have to have CT 3D recons demonstrating a smooth inward deformity. You
could never see that shit on a plain film. I can’t imagine anyone being a big enough
asshole to ask you to do that. Hmmm…. probably.

How well did you know this?
1
Not at all
2
3
4
5
Perfectly
45
Q

Ping Pong Fracture

A

This is actually another subtype of depressed fracture but is unique in that it is
a greenstick or “buckle” type of fracture.

How well did you know this?
1
Not at all
2
3
4
5
Perfectly
46
Q

NEXT STEP

Depressed Fx

A

Unlike linear fractures (which usually heal without complication),
depressed fractures often require surgery. Some general indications Fracture
for surgery would include:

Depression of the fragments > 5mm (supposedly fragments more
than 5mm below the inner table are associated with dural tears),
Epidural bleed
Superinfection (abscess, osteomyelitis)
“Form” (cosmetic correction to avoid looking like a gargoyle),
“Function” (if the frontal sinus is involved, sometimes they need
to obliterate the thing to avoid mucocele formation).

How well did you know this?
1
Not at all
2
3
4
5
Perfectly
47
Q

Non-Accidental Trauma t Abusive Head Trauma)

A
Although car wrecks and falls account for the majority o f skull fractures in children, there
still remains the timeless truth — some people just can’t take screaming kids.
For the purpose of multiple choice, the follow clues should make your spider-sense tingle.
• Inconsistent History:
“My 7 month old wrecked his bike ”
• Subdural Hematoma
• Retinal hemorrhage
• DAI / Parenchymal Contusion
• Cerebral Edema, Stroke
(less specific but still worrisome)
• Depressed Skull Fx, or Fracture
Crossing Suture Line
(less specific but still worrisome)
How well did you know this?
1
Not at all
2
3
4
5
Perfectly
48
Q

Non-Accidental Trauma t Abusive Head Trauma)

gamesmansship

A

Subdurals have a stronger association with NAT
relative to epidurals. Think about vigorous
shaking (trying to get that last drop of ketchup
out of the bottle) tearing bridging cortical veins.
“Look High, Look Low”
- Sneaky Ways to Suggest NAT -
- Look High: Thrombosed (hyperdense)
cortical vein at the vertex
- Look Low: Retroclival hematoma (thin
hyperdense sliver in the pre-pontine region)
- Look Lower: Edema within the cervical soft
tissues

How well did you know this?
1
Not at all
2
3
4
5
Perfectly
49
Q

THIS VS THAT:

Retroclival Hematoma

A

Epidural
Below the Tectorial
Membrane

Subdural
Above the tectorial Membrane

How well did you know this?
1
Not at all
2
3
4
5
Perfectly
50
Q

THIS vs THAT:

Extra Axial Fluid

A

Chronic Subdural
(CSF Density)

Medial Displacement
of Bridging Vein
(sometimes smashed and
not well seen).

Usually Unilateral.

If Bilateral Usually
Asymmetric in Size.

Prominent CSF Spaces

Cortical veins are
adjacent to the inner table

Usually Symmetric

How well did you know this?
1
Not at all
2
3
4
5
Perfectly
51
Q

Enlarged extra-axial fluid spaces

A

Extra-axial fluid spaces are considered enlarged if they are
greater than 5 mm. BESSI is the name people throw around
for “benign enlargement of the subarachnoid space in infancy.’
The etiology is supposed to be immature villa (that’s why you
grow out of it).

How well did you know this?
1
Not at all
2
3
4
5
Perfectly
52
Q

THIS vs THAT:

BESSI vs Subdural Hygroma

A
BESSI - Cortical veins are
adjacent to the inner table - they
are usually seen secondary to
enlargement o f the subarachnoid
spaces (positive cortical vein
sign)
Subdural - Cortical veins are
displaced away from the inner
table - they are often not se
How well did you know this?
1
Not at all
2
3
4
5
Perfectly
53
Q

BESSI Trivia:

A

• It’s the most common cause of macrocephaly,
• Typically presents around month 2 or 3, and has a strong
male predominance.
• Typically resolves after 2 years with no
treatment,
• There is an increased risk of subdural bleed -
either spontaneous or with a minor trauma. This
subdural is usually isolated (all the same blood
age), which helps differentiate it from nonaccidental
trauma, where the bleeds are often of
different ages.

How well did you know this?
1
Not at all
2
3
4
5
Perfectly
54
Q

Enlarged extra-axial fluid spaces

trivia

A
Pre-mature kids getting tortured
on ECMO often get enlarged extra-axial
spaces. This isn’t really the same thing
as BESSI but rather more related to fluid
changes / stress.
How well did you know this?
1
Not at all
2
3
4
5
Perfectly
55
Q

Bessi imaging

A

Enlarged symmetric subarachnoid spaces favoring the anterior aspect o f the brain (spaces along the posterior aspect o f the brain are typically normal).

Brain parenchyma is normal and there is either normal ventricle size or very mild
communicating hydrocephalus. Communicating meaning that all 4 ventricles are big.

How well did you know this?
1
Not at all
2
3
4
5
Perfectly
56
Q

Periventricular leukomalacia I Hypoxic-Ischemic Encephalopathy of the Newborn

overview

A

This is the result of an ischemic / hemorrhagic injury, typically from a hypoxic insult during
birthing. The kids who are at the greatest risk are premature and little (less than 1500 g). The
testable stigmata is cerebral palsy - which supposedly develops in 50%. The pathology favors
the watershed areas (characteristically the white matter dorsal and lateral to the lateral
ventricles).

How well did you know this?
1
Not at all
2
3
4
5
Perfectly
57
Q

Periventricular leukomalacia I Hypoxic-Ischemic Encephalopathy of the Newborn

The milder finding can be very subtle. Here are some tricks:

A

The milder finding can be very subtle. Here are some tricks:

(1) Use PreTest Probability: The kid is described as premature or low birth weight.

(2) Brighter than the Choroid: The choroid plexus is an excellent internal control. The
normal white matter should always be less bright (less hyperechoic) when compared to the choroid

(3) “Blush” and “Flaring” : These are two potential distractors that need to be differentiated
from legit grade 1 PVL. “Blush ” describes the physiologic brightness of the
posterosuperior periventricular white matter - this should be less bright than choroid, and
have a more symmetric look. “Flaring” is similar to blush, but a more hedgy term. It’s the
word you use if you aren’t sure if it’s real PVL or just the normal brightness often seen in
premature infants white matter. The distinction is that “flaring” should go away in a
week. Grade 1 PVL persists > 7 days.

How well did you know this?
1
Not at all
2
3
4
5
Perfectly
58
Q

Periventricular leukomalacia I Hypoxic-Ischemic Encephalopathy of the Newborn

quick

A

Early: P e riv en tric u la r W h ite M a tte r N e c ro sis
(hyperechoic relative to choroid)

S u b acu te: C y s t F o rm a tio n

How well did you know this?
1
Not at all
2
3
4
5
Perfectly
59
Q

Periventricular leukomalacia I Hypoxic-Ischemic Encephalopathy of the Newborn

trivia

A

The most severe grade (4), which has subcortical cysts, is actually more common in
full term infants rather than preterms.

How well did you know this?
1
Not at all
2
3
4
5
Perfectly
60
Q

Periventricular leukomalacia I Hypoxic-Ischemic Encephalopathy of the Newborn

A

The later findings are more obvious with the development of cavitary periventricular cysts.
The degree of severity is described by the size and distribution of these cysts. These things take
a while to develop - some people say up to 4 weeks. So, if they show you a day 1 newborn
with cystic PVL they are leading you to conclude that the vascular insult occurred at least 2
weeks prior to birth (not during birth - which is often the case).

How well did you know this?
1
Not at all
2
3
4
5
Perfectly
61
Q

Germinal Matrix Hemorrhage ( GMH)

A

I like to think about the germinal matrix as an embryologic seed that sprouts out various
development cells during brain development. Just like a seed needs water to grow, the
germinal matrix is highly vascular. It’s also very friably and susceptible to stress.
Additionally, premature brains suck at cerebral blood flow auto-regulation. Mechanism:
Fragile vessels + too much pressure/flow = bleeds
An important thing to understand is that the germinal matrix is an embryological entity. So it
only exists in premature infants. As the fetus matures the thing regresses and disappears.
By 32 weeks, germinal matrix is only present at the caudothalamic groove.
By 36 weeks, you basically can’t have it (if no GM, then no GM hemorrhage).

How well did you know this?
1
Not at all
2
3
4
5
Perfectly
62
Q

Germinal Matrix Hemorrhage ( GMH)

Take home point

A

No GM Hemorrhage in a full term infant.

How well did you know this?
1
Not at all
2
3
4
5
Perfectly
63
Q

Germinal Matrix Hemorrhage ( GMH)

Gamesmanship

A

Similar looking bleed in a full term infant say “choroid plexus
hemorrhage” (not GMH).

How well did you know this?
1
Not at all
2
3
4
5
Perfectly
64
Q

Germinal Matrix Hemorrhage ( GMH)

scenario

A

The scenario will always call the kid a premature infant (probably earlier than 30 weeks). The
earlier they are bom the more common it is. Up to 40% occur in the first 5 hours, and most
have occurred by day 4 (90%). A good thing to remember is that 90% occur in the first week.

How well did you know this?
1
Not at all
2
3
4
5
Perfectly
65
Q

Germinal Matrix Hemorrhage ( GMH)

screening

A

Head US is used to screen for this pathology. Testable trivia includes:
• Who should be screened? Premature Infants (<32 weeks, < 1500 grams), Premature Infants
with Lethargy, Seizures, Decreased Hematocrit or a history o f “he don’t look so good.”
• When do you do the head US? First week o f life (remember this is when 90% o f them occur).
Some people will tell you - “first week and first month” (but that varies from institution).
Some people will also say - “every kid gets a head US prior to discharge from the NICU” - but
that is mainly done to detect PVL (not necessarily GMH).

How well did you know this?
1
Not at all
2
3
4
5
Perfectly
66
Q

Germinal Matrix Vs Choroid Plexus

A

Choroid Plexus is bright (hyperechoic) on ultrasound. Blood is also bright (hyperechoic).

You tell them apart based on their location. Choroid should not extend anterior to the junction o f the caudate and the thalamus (the so called caudothalamic groove).

This is the location o f the germinal matrix.

If you see bright stuff there - that is your grade 1 bleed.

This could also be shown with MR1 (T1 bright = bleed), same location.

How well did you know this?
1
Not at all
2
3
4
5
Perfectly
67
Q

Germinal Matrix Hemorrhage ( GMH)

Grading System (1-4) -

A
1 - Blood at the caudothalamic groove
2 - Blood in the ventricles
but no dilation.
3 - Blood in the ventricles with dilation
4 - Blood in the brain parenchyma
(from venous infarct)
How well did you know this?
1
Not at all
2
3
4
5
Perfectly
68
Q

Few additional things you should see at least once
prior to the exam. Do yourself a favor and
google images o f the following

A
  • Caudothalamic groove in the coronal plane
  • Grade 2,3,4 Bleeds
  • Subependymal Cysts on US
  • Porencephalic Cysts on US
  • Choroid Plexus Cysts on US
  • Ventricular coarctation on US - this is a mimic
How well did you know this?
1
Not at all
2
3
4
5
Perfectly
69
Q

Secondary Consequences GMH

Grade 1

A

Subependymal hemorrhage either results in a subependymal cyst or resolution or to a grade 2

How well did you know this?
1
Not at all
2
3
4
5
Perfectly
70
Q

Secondary Consequences GMH

Grade 2/3

A

Intraventricular hemorrhage results in hydrocephalus or grade 4

How well did you know this?
1
Not at all
2
3
4
5
Perfectly
71
Q

Secondary Consequences GMH

Grade 4

A

intraparenchymal hemorrhage either goes to a porencephalic cyst or hydrocephalus.

How well did you know this?
1
Not at all
2
3
4
5
Perfectly
72
Q

Choanal Atresia

A

Results from a membrane that separates the nasal cavity from its normal communication with the oral cavity. It is
usually unilateral but can be bilateral. The bilateral ones have a history
of “cyclical cyanosis that improves with crying ” (they mouth breath
when they cry). The unilateral classic stories are “can’t pass NG
tube, ” and “respiratory distress while feeding” (neonates have to
breath through their noses). You will also sometimes hear a history of
“a continuous stream o f snot draining from one or both nostrils, ” or
the word “rhinorrhea. ”

How well did you know this?
1
Not at all
2
3
4
5
Perfectly
73
Q

Choanal Atresia

types

A

There are two different types: bony (90%), and
membranous (10%). The appearance is a
unilateral or bilateral posterior nasal narrowing,
with thickening of the vomer.

How well did you know this?
1
Not at all
2
3
4
5
Perfectly
74
Q

Choanal Atresia

trivia

A

There are many syndromic associations
including CHARGE. Crouzons, DiGeorge,
Treacher Collins, and Fetal Alcohol Syndrome.
CHARGE is the one people mention the most.

How well did you know this?
1
Not at all
2
3
4
5
Perfectly
75
Q

CHARGE

A
Coloboma,
Heart defect,
Atresia (Choanal)
Retarded growth,
Genitourinary abnormalities
Ear anomalies
How well did you know this?
1
Not at all
2
3
4
5
Perfectly
76
Q

Congenital Piriform Aperture Stenosis

A

This results from abnormal development of the medial nasal eminences,
and subsequent failure of formation of the primary palate. You can
see this in isolation or with choanal atresia. The piriform aperture of
the nasal cavity (bony inlet of the nose) is stenotic (as the name
suggests), and the palate is narrow. The classic picture is the
associated central maxillary “MEGA-incisor.” Midlinc defects
of the brain (corpus callosum agenesis, and holoprosencephaly) are
associated
— as my Grandma always said “face predicts brain ”.

How well did you know this?
1
Not at all
2
3
4
5
Perfectly
77
Q

Congenital Piriform Aperture Stenosis

next step

A

You have to image the brain

How well did you know this?
1
Not at all
2
3
4
5
Perfectly
78
Q

Congenital Piriform Aperture Stenosis

big thing to know

A

The big thing to know is the high association with hypothalamicpituitary-
adrenal axis dysfunction.

How well did you know this?
1
Not at all
2
3
4
5
Perfectly
79
Q

Ectopic Thyroid

A

Thyroid topics will be covered again in the endocrine chapter. 1 do want to mention one or two now for
completeness. To understand ectopic thyroid trivia you need to remember that the thyroid starts
(embryology wise) at the back of the tongue. It then descends downward to a location that would be
considered normal. The “pyramidal lobe” actually represents a persistence of the inferior portion of the
thyroglossal duct - that is why this thing is so variable in appearance. Sometimes this process gets all
fucked up and the thyroid either stays at the back of the tongue (lingual thyroid) or ends up half way
down the neck or even in the chest (ectopic thyroid).

How well did you know this?
1
Not at all
2
3
4
5
Perfectly
80
Q

Ectopic Thyroid

trivia to know

A
  • Most “developed” countries test for low thyroid at birth (Guthrie Test). That will trigger a workup for either ectopic tissue or enzyme deficiencies.
  • Nukes (1-123 or Tc-MIBI) is superior to ultrasound for diagnosing ectopic tissue. This is by far the most likely way to show this on a multiple choice exam. I guess CT would be #2 - remember thyroid tissue is dense because of the iodine.
  • Ultrasound does have a preoperative role in any MIDLINE neck mass - with the point of ultrasound being to confirm that you have a normal thyroid in a normal place. If you resect a midline mass (which turns out to be the kids only thyroid tissue) you can expect an expensive well rehearsed didactic lecture on pediatric neck pathology from an “Expert Witness” sporting a $500 haircut.
  • Lingual thyroid (back of the tongue) is the most common location of ectopic thyroid tissue
How well did you know this?
1
Not at all
2
3
4
5
Perfectly
81
Q

Thyroglossal Duct Cyst

A

As we discussed previously, thyroid related pathology can occur
anywhere between the foramen cecum (the base of the tongue)
and the thyroid gland. In this situation we are talking about the
duct (which is the embryological thyroid interstate highway to the
neck) failing to involute fully. What you get is a left over cyst -
hence the name. The classic locations are (1) at the base of the
tongue, and (2) midline anterior to the hyoid. Now textbooks will
make a big deal about these things becoming slightly lateral below
the hyoid. Do NOT get hung up on that. For the purpose of multiple
choice remember these guys are midline. Midline is the buzzword.

How well did you know this?
1
Not at all
2
3
4
5
Perfectly
82
Q

Thyroglossal Duct Cyst

things to know

A
  • Classic Buzzword / Scenario = Midline Cyst in the Neck o f a Kid.
  • Next step once you find one = confirm normal thyroid location and/or look for ectopic tissue (Ultrasound +/-Tc-M1BI, or 1-123).
  • They are cystic (it’s not called a “Duct Solid”)
  • Enhancing nodule within the cyst = CANCER (usually papillary)
  • They can get infected.
How well did you know this?
1
Not at all
2
3
4
5
Perfectly
83
Q

Dermoid Cyst

A

It is true that dermoids almost always occur below the clavicles, but
when they do happen in the neck they have a pretty classic look:
midline sublingual / submandibular space with a “sac of marbles”
appearance. The marbles are lobules of fat within fluid.

How well did you know this?
1
Not at all
2
3
4
5
Perfectly
84
Q

Branchial Cleft Cyst (BCC)

A

Another cystic embryologic remnant. There are a bunch o f types (and subtypes… and subsubtypes)
and you can lose your fucking mind trying to remember all o f them - don’t do that.
Just remember that by far the most common is a 2nd Branchial Cleft Cyst (95%). The angle
of the mandible is a classic location. They can get infected, but are often asymptomatic.
Extension o f the cyst between the ICA and ECA (notch sign) just above the carotid
bifurcation is pathognomonic.

How well did you know this?
1
Not at all
2
3
4
5
Perfectly
85
Q

Branchial Cleft Cyst (BCC)

what is it

A

Most likely on CT or MR1. Ultrasound would be tough, unless they clearly
labeled the area “lateral neck” or oriented you in some other way.

How well did you know this?
1
Not at all
2
3
4
5
Perfectly
86
Q

Branchial Cleft Cyst (BCC)

location

A
They could (and this would be super mean) ask you the relationship o f a
type 2 based on other neck anatomy. So - posterior and lateral to the submandibular gland
or lateral to the carotid space, or anterior to the sternocleidomastoid. How I would handle
that? Just remember it’s going to be lateral to everything. Lateral is the buzzword
How well did you know this?
1
Not at all
2
3
4
5
Perfectly
87
Q

Branchial Cleft Cyst (BCC)

mimic

A
They could try and trick you into calling a necrotic level 2 lymph node a BCC.
Thyroid cancer (history o f radiation exposure) and nasopharyngeal cancer (history ofHPV)
can occur in “early adulthood.” If you have a “new” BCC in an 18 year old - it’s probably a
necrotic node. Next Step = Find the cancer +/- biopsy the mother fucker.
How well did you know this?
1
Not at all
2
3
4
5
Perfectly
88
Q

I say LATERAL
cyst in the neck,
you say

A

branchial cleft cyst

How well did you know this?
1
Not at all
2
3
4
5
Perfectly
89
Q

1 say MIDLINE
cyst in the neck,
you say

A

thyroglossal duct cyst

How well did you know this?
1
Not at all
2
3
4
5
Perfectly
90
Q

Jugular Vein Pathology

Septic Thrombophlebitis

A

clotted jugular vein. You see this classically in the setting of a
recent pharyngeal infection (or recent ENT surgery).

How well did you know this?
1
Not at all
2
3
4
5
Perfectly
91
Q

Jugular Vein Pathology

Septic Thrombophlebitis

what is it

A

Showing the clotted vein with the appropriate clinical history.

How well did you know this?
1
Not at all
2
3
4
5
Perfectly
92
Q

Jugular Vein Pathology

Septic Thrombophlebitis

“Lemierre’s Syndrome”

A

Seeing if you know that it has a fancy syndrome name.

How well did you know this?
1
Not at all
2
3
4
5
Perfectly
93
Q

Jugular Vein Pathology

Septic Thrombophlebitis

Next Step

A

Looking in the lungs for septic emboli. This could also be done in the reverse. Show
you the septic emboli, give you a history of ENT procedure (or recent infection), and have you ask
for the US of the neck veins.

How well did you know this?
1
Not at all
2
3
4
5
Perfectly
94
Q

Jugular Vein Pathology

Septic Thrombophlebitis

USMLE Step I Association Trivia

A

Fusobacterium necrophorum is the bacteria that causes the
septic emboli. As this bacteria sounds like a Marvel Comic villain the likelihood of it being asked
increases by at least 5x.

How well did you know this?
1
Not at all
2
3
4
5
Perfectly
95
Q

Jugular Vein Pathology

Phlebectasia

A

Idiopathic dilated jugular vein.

How well did you know this?
1
Not at all
2
3
4
5
Perfectly
96
Q

Jugular Vein Pathology

Phlebectasia

What is it

A

showing the dilated vein

How well did you know this?
1
Not at all
2
3
4
5
Perfectly
97
Q

Jugular Vein Pathology

Phlebectasia

trivia 1

A

is it not related to a stenosis. there aren o other signs of venous congestion

How well did you know this?
1
Not at all
2
3
4
5
Perfectly
98
Q

Jugular Vein Pathology

Phlebectasia

trivia 2

A

It gets worse with the Valsalva maneuver - “neck mass that enlarges with valsalva. ”

How well did you know this?
1
Not at all
2
3
4
5
Perfectly
99
Q

Both lymphatic and venous malformations can both look like a large transspatial
multicystic mass in the neck. They can both have fluid levels.

A

If you must try and tell them apart - you could try this:
• Venous Malformations will have enhancement of the cystic spaces.
• Lymphatic Malformations will have enhancement of the septa.
• Phlcboliths — suggests venous.

How well did you know this?
1
Not at all
2
3
4
5
Perfectly
100
Q

Hemangioma of Infancy

A

These things are actually the most common congenital lesions in the head and neck. Just like
any hemangioma they contain vascular spaces with varying sizes and shapes. Most people
consider them a “tumor” more than a vascular malformation.

How well did you know this?
1
Not at all
2
3
4
5
Perfectly
101
Q

Hemangioma of Infancy

how they look

A

Super T2 bright, with a bunch of

flow voids. Diffusely vascular on doppler.

How well did you know this?
1
Not at all
2
3
4
5
Perfectly
102
Q

Hemangioma of Infancy

phases

A

Typically they show up around 6 months of age, grow for a bit, then plateau, then involute (6-10 years). Usually they require no treatment.

How well did you know this?
1
Not at all
2
3
4
5
Perfectly
103
Q

Hemangioma of Infancy

indications for treatment

A

Large size / Rapid growth

with mass effect on the airway or adjacent vascular structures. Fucking with the kids eye movement or eyelid opening.

How well did you know this?
1
Not at all
2
3
4
5
Perfectly
104
Q

Hemangioma of Infancy

treatment

A

Typically medical = Beta blocker

propranolol

How well did you know this?
1
Not at all
2
3
4
5
Perfectly
105
Q

Hemangioma of Infancy

associations

A
PHACES Syndrome (discussed later
in the chapter)
How well did you know this?
1
Not at all
2
3
4
5
Perfectly
106
Q

Cystic Hygroma (Lymphangioma)

A

This is another cystic lesion o f the neck, which is most likely to be shown as an OB
ultrasound (but can occur in the Peds setting as well). The classic look / location is a cystic
mass hanging off the back o f the neck on OB US (or in the posterior triangle if CT/MR1).

How well did you know this?
1
Not at all
2
3
4
5
Perfectly
107
Q

Cystic Hygroma (Lymphangioma)

associations

A

Turners (most common association).

Downs (second most common association).

Aortic Coarctation (most common
CV abnormality), 
Fetal Hydrops (bad bad bad outcomes).
How well did you know this?
1
Not at all
2
3
4
5
Perfectly
108
Q

Cystic Hygroma (Lymphangioma)

septations

A

worse outcome

How well did you know this?
1
Not at all
2
3
4
5
Perfectly
109
Q
Cystic Hygroma (Lymphangioma)
t2
A
T2 Bright (like a hemangioma).
Does NOT enhance (hemangiomas
typically do).
How well did you know this?
1
Not at all
2
3
4
5
Perfectly
110
Q

Fibromatosis Coli (“Congenital Torticollis”)

overview

A

This is a benign “mass” of the sternocleidomastoid
in neonates who present with torticollis (chin points
towards the opposite side - or you could say they
look away from the lesion). It’s really just a benign
inflammation that makes the muscle look crazy big.
Ultrasound can look scary, until you realize it’s just
the enlarged SCM. Ultrasound is still the best
imaging test. Sometimes it looks like there are two
of them, but that’s because the SCM has two heads.
It goes away on its own, sometimes they do passive
physical therapy or try and botox them.

How well did you know this?
1
Not at all
2
3
4
5
Perfectly
111
Q

Fibromatosis Coli (“Congenital Torticollis”)

trivia

A

• Most common cause of a neck “mass” in infancy
• Classic scenario is a 4 week old with a palpable neck mass and torticollis toward the affected side
• Best imaging test = US.
• Things that make you think it’s not FC: mass is outside the SCM, or internal calcifications - in
which case you should think to yourself… nice try Mother fuckers- that’s a neuroblastoma.

How well did you know this?
1
Not at all
2
3
4
5
Perfectly
112
Q

Fibromatosis Coli (“Congenital Torticollis”)

gamesmanship

A

So… there can be significant fuckery with the “direction” things curve or people look depending
on how the question is ask. What do I mean ?
If you made the mistake of just memorizing the word “towards” in association with fibromatosis
coli you might get tricked if the options were: A - Patient looks towards the involved side.
B - Patient looks toward the uninvolved side. You’d run into the same problem with the word
“away.”
Now, that might seem obvious once I spell it out like that but I’m pretty sure at least a few of you
were making a flashcard that had only the word “towards” on it. You have to assume the test
writer has the worst intentions for you. Don’t provide them with any opportunity to trick you.

How well did you know this?
1
Not at all
2
3
4
5
Perfectly
113
Q

Rhabdomyosarcoma

A

Although technically rare as fuck, this is the most common mass in the masticator space of a kid.
Having said that if you see it in the head/neck region is almost always in the orbit. In fact, its the most
common extra-occular orbital malignancy in children (dermoid is most common benign orbital mass
in child). The most classic scenario would be an 8 year old with painless proptosis and no signs of
infection.

How well did you know this?
1
Not at all
2
3
4
5
Perfectly
114
Q

What do sarcomas look like?

A

I’ll talk about this more in the MSK chapter, but in general I’ll just say they look mean as cat shit
(enhancing, solid, areas of necrosis, etc..).

How well did you know this?
1
Not at all
2
3
4
5
Perfectly
115
Q

Croup

A

This is the most common cause o f acute upper airway obstruction in young children. The
peak incidence is between 6 months and 3 years (average 1 year). They have a barky
“croupy” cough. It’s viral. The thing to realize is that the lateral and frontal neck x-ray is
done not to diagnosis croup, but to exclude something else. Having said that, the so-called
“steeple sign ” - with loss o f the normal lateral convexities o f the subglottic trachea is your
buzzword, and if it’s shown, that will be the finding. Questions are still more likely to center
around facts (age and etiology).

How well did you know this?
1
Not at all
2
3
4
5
Perfectly
116
Q

Croup

culprit

A

The culprit is often parainfluenza virus.

How well did you know this?
1
Not at all
2
3
4
5
Perfectly
117
Q

Epiglottitis

A

In contrast to the self-limited croup, this one can kill you. It’s mediated by H. Influenza and
the classic age is 3.5 years old (there is a recent increase in teenagers - so don’t be fooled by
that age). The lateral x-ray will show marked swelling o f the epiglottis (thumb sign). A fake
out is the “omega epiglottis” which is caused by oblique imaging. You can look for thickening
o f the aryepiglottic folds to distinguish.

How well did you know this?
1
Not at all
2
3
4
5
Perfectly
118
Q

Epiglottitis

trivia

A

Death by asphyxiation is from the aryepiglotic folds (not the epiglottis)

How well did you know this?
1
Not at all
2
3
4
5
Perfectly
119
Q

Exudative Tracheitis (Bacterial Tracheitis)

A

This is an uncommon but serious (possibly deadly) situation that is found in slightly older
kids. It’s caused by an exudative infection o f the trachea (sorta like diptheria). It’s usually
from Staph A. and affects kids between 6-10.

How well did you know this?
1
Not at all
2
3
4
5
Perfectly
120
Q

Exudative Tracheitis (Bacterial Tracheitis)

buzzword

A

linear soft tissue filling defect

within the airway.

How well did you know this?
1
Not at all
2
3
4
5
Perfectly
121
Q

Croup

quick

A

6 months - 3 years
(peak 1 year)

Steeple Sign: loss o f the normal
shoulders (lateral convexities) of
the subglottic trachea

Viral
(Most Common - parainfluenza)

How well did you know this?
1
Not at all
2
3
4
5
Perfectly
122
Q

epiglottitis quick

A

Classic = 3.5 years, but now
seen with teenagers too

Thumb Sign: marked
enlargement o f epiglottis

h. flu

How well did you know this?
1
Not at all
2
3
4
5
Perfectly
123
Q

Exudative tracheitis quick

A

6-10 years

Linear soft tissue filling
defect (a membrane) seen
within the airway

Staph A

How well did you know this?
1
Not at all
2
3
4
5
Perfectly
124
Q

Retropharyngeal Cellulitis and Abscess

A

I’ll just say quickly that you do see this
most commonly in young kids (age 6 months -12 months). If they don’t show it on CT, they
could show it with a lateral x-ray demonstrating massive retropharyngeal soft tissue
thickening. For the real world, you can get pseudothickcning when the neck is not truly lateral
To tell the difference between positioning and the real thing, a repeat with an extended neck is
the next step.

How well did you know this?
1
Not at all
2
3
4
5
Perfectly
125
Q

Subglottic Hemangioma

A

Hemangiomas are the most common soft tissue mass in the
trachea, and they are most commonly located in the subglott
region. In croup there is symmetric narrowing with loss of
shoulders on both sides (Steeple Sign). In contradistinction,
subglottic hemangiomas have loss of just one o f the sides.

How well did you know this?
1
Not at all
2
3
4
5
Perfectly
126
Q

Subglottic Hemangioma

Trivia

A
  • Tends to favor the left side
  • 50% are associated with cutaneous hemangiomas
  • 7% have the PHACES syndrome
How well did you know this?
1
Not at all
2
3
4
5
Perfectly
127
Q

PHACES

A
P- Posterior fossa
(Dandy Walker)
H- Hemangiomas
A- Arterial anomalies
C- Coarctation o f aorta,
cardiac defects
E- Eye abnormalities
S- Subglottic
hemangiomas
How well did you know this?
1
Not at all
2
3
4
5
Perfectly
128
Q

Laryngeal Cleft

A

This is a zebra. The classic scenario is contrast appearing in the tracheal without laryngeal penetration
(aspiration). They could also show you a “thin tract of contrast extending to the larynx or trachea.” This
entity is a communicating defect in the posterior wall of the larynx and the esophagus or anterior hypo
pharynx. There arc a bunch of different cleft classifications - 1 can’t imagine that shit is appropriate for
the exam.

How well did you know this?
1
Not at all
2
3
4
5
Perfectly
129
Q

Laryngeal Cleft

trivia

A

• This is a thing - maybe google a Fluoro swallow picture of it
• These things have other complex malformations associated with them (usually GI)
• It is very tricky to call it with certainty on a swallow exam - definite diagnosis is always made with
direct visualization / scope (so a next step type question would recommend endoscopy to confirm).

How well did you know this?
1
Not at all
2
3
4
5
Perfectly
130
Q

airway papilloma

A

If you see a lobulated grape looking thing in the airway - think Papilloma, especially if the lungs are full of nodules (solid and cavitated). When 1 say Papilloma, You say HPV - typically from
perinatal (birth canal) transmission. These things are usually multiple (papillomatosis) and therefore have
multiple areas of airspace disease (atelectasis ect.). Some potential gamesmanship — because these thing
are typically multiple you will have more areas of air trapping then you would compared with an
aspirated crayon (or green bean), or even a solitary endobronchial lesion like a carcinoid. Multiple areas
of air trapping - think Papillomatosis over carcinoid or a foreign body. Having said that the nodules are a
more common finding… lots of them.

How well did you know this?
1
Not at all
2
3
4
5
Perfectly
131
Q
  • G am e sm a n s h ip -
    F ro n ta l an d L a te ra l N e c k R a d io g ra p h s

For the frontal, there are
two main things to think
about.

A

(1) Croup and
(2) Subglottic
Hemangioma
You can tell them apart
by the shouldering.
If you can’t tell…. try
and let the history bias
you. Cough? Fever?
Think Croup.

How well did you know this?
1
Not at all
2
3
4
5
Perfectly
132
Q
  • G am e sm a n s h ip -
    F ro n ta l an d L a te ra l N e c k R a d io g ra p h s

For the lateral, there are 4 main things to think about:

A

epiglottitis
retropharygeal abscess
tonsis (adenoids)
exudative tracheitis

How well did you know this?
1
Not at all
2
3
4
5
Perfectly
133
Q

lateral radiograph

epiglottitis

A

Looks like a
thumb

If the ordering
suspects the
diagnosis, do
NOT bring this
kid to x-ray. Have
them do a
portable.
How well did you know this?
1
Not at all
2
3
4
5
Perfectly
134
Q

lateral radiograph

retropharyngeal abscess

A
Too wide
( > 6mm at C2,
or > 22mm at C6)
Next Step = CT
don't forget to look
in the mediastinum
for “Danger Zone”
extension.
How well did you know this?
1
Not at all
2
3
4
5
Perfectly
135
Q

lateral radiograph

tonsils (adenoids)

A

Not seen till about
3-6 months, and not
big till around 1-2
years.

Too big when they
encroach the airway

How well did you know this?
1
Not at all
2
3
4
5
Perfectly
136
Q

lateral radiograph

exudative tracheitis

A

Linear Filling
Defect

It’s usually staph

How well did you know this?
1
Not at all
2
3
4
5
Perfectly
137
Q

Know how to tell if a neonatal chest is hyper-inflated or not. Don’t get hung up on this
low vs normal - that’s a bunch o f bologna. Just think (a) Hyper-inflated, or (b) NOT
Hyper-inflated.

A

The easiest way to do this is to just count ribs. More than 6 Anterior, or 8 Posterior as they intersect the diaphragm is too much. As a quick review, remember that the anterior ribs (grey) are the ones with a more sloping course as they move medially, where as the posterior ribs (black) have a horizontal course.

How well did you know this?
1
Not at all
2
3
4
5
Perfectly
138
Q

Other helpful signs suggesting

hyperinflation:

A
  • Flattening o f the diaphragms

* Ribs take on a more horizontal appearance

How well did you know this?
1
Not at all
2
3
4
5
Perfectly
139
Q

Know what “Granular” looks like. Know what “Streaky” or “Ropy” looks like. My
good friends at Amazon are not capable of printing a clear picture o f these so I want you
to stop reading and

A

A. Go to google images
B. Search “Granular neonatal chest x-ray.’” Look at a bunch o f examples. Maybe even
download a few of them for review.
C. Search “Streaky Perihilar neonatal chest x-ray.” Look at a bunch o f examples.
Maybe even download a few o f them for review.
D. Search “Ropy neonatal chest x-ray.” Look at a bunch o f examples. Maybe even
download a few o f them for review.

How well did you know this?
1
Not at all
2
3
4
5
Perfectly
140
Q

Chest

random pearl

A

We are going to talk about the presence o f a pleural effusion as a
discriminator. One pearl is to look fo r an accentuated (thick) minor fissure on the right. If
you see that shit, kid probably has an effusion. Confirm by staring with fierce intensity at the
lung bases to look for obliteration o f the costophrenic sulcus.

How well did you know this?
1
Not at all
2
3
4
5
Perfectly
141
Q

High Volumes

+ Perihilar Streaky

A

Alphabet - MNoP
Meconium Aspiration
Non GB Neonatal
Pneumonia

How well did you know this?
1
Not at all
2
3
4
5
Perfectly
142
Q

Not High (low or
normal) Volumes
+ Granular

A

SSD

Group B Pneumonia

How well did you know this?
1
Not at all
2
3
4
5
Perfectly
143
Q

Meconium Aspiration

A

This typically occurs secondary to stress (hypoxia), and is more common in term or postmature
babies (the question stem could say “post term ” delivery). The pathophysiology is all
secondary to chemical aspiration.

How well did you know this?
1
Not at all
2
3
4
5
Perfectly
144
Q

Meconium Aspiration

trivia

A
  • The buzzword “ropy appearance” o f asymmetric lung densities
  • Hyperinflation with alternative areas o f atelectasis
  • Pneumothorax in 20-40% o f cases
How well did you know this?
1
Not at all
2
3
4
5
Perfectly
145
Q

Meconium Aspiration

hyperinflation

A

How can it have hyperinflation?? Aren V the lungs fu l l o f sticky shit
(literally) ??? The poop in the lungs act like miniature bal 1-valves
(“floaters” 1 call them), causing air trapping - hence the increased lung
volumes.

How well did you know this?
1
Not at all
2
3
4
5
Perfectly
146
Q

Meconium Aspiration

reality vs multiple choice

A

“Meconium Staining ” on the amniotic fluid is common (like 15% o f all
births), but development o f “aspiration syndrome” is rare with only 5% o f those 15% actually have
aspiration symptoms. Having said that, if the question header bothers to include “Green colored
amniotic fluid” or “Meconium staining” in the question header they are giving you a major hint.
Don’t overthink a hint like this. I f they ask “What color was George Washington s white horse ? ” the
answer is NOT brown.

How well did you know this?
1
Not at all
2
3
4
5
Perfectly
147
Q

Transient Tachypnea of the Newborn (TTN)

A

The classic clinical scenario is a history o f c-section (vagina squeezes the fluid out of lungs
normally). Other classic scenario histories include “diabetic mother” and/or “maternal
sedation.” Findings are going to start at 6 hours, peak at one day, and be done by 3 days. You
are going to see coarse interstitial marking and fluid in the fissures.

How well did you know this?
1
Not at all
2
3
4
5
Perfectly
148
Q

Transient Tachypnea of the Newborn (TTN)

trivia

A
  • Classic histories: C-Section, Maternal Sedation, Maternal Diabetes
  • Onset: Peaks at day 1, Resolved by Day 3
  • Lung Volumes - Normal to Increased
How well did you know this?
1
Not at all
2
3
4
5
Perfectly
149
Q

Surfactant-Deficient Disease (SDD)

A

This is also called hyaline membrane disease, or RDS. It’s a disease of pre-mature kids. The idea
is that they are bom without surfactant (the stuff that makes your lungs stretchy and keeps alveolar
surfaces open). It’s serious business and is the most common cause of death in premature
newborns. You get low lung volumes and bilateral granular opacities (just like B-hcmolytic
pneumonia). But, unlike B-hemolytic pneumonia you do NOT get pleural effusions. As a piece
of useful clinical knowledge, a normal plain film at 6 hours excludes SDD.

How well did you know this?
1
Not at all
2
3
4
5
Perfectly
150
Q

Surfactant Replacement Therapy

A

They can spray this crap in the kid’s lungs, and it makes a huge difference (decreased death rate
etc…). Lung volumes get better, and granular opacities will clear centrally after treatment. The
post treatment look of bleb-like lucencies can mimic PIE.

How well did you know this?
1
Not at all
2
3
4
5
Perfectly
151
Q

Surfactant Replacement Therapy

trivia

A
  • Increased Risk of Pulmonary Hemorrhage

* Increased Risk of PDA

How well did you know this?
1
Not at all
2
3
4
5
Perfectly
152
Q

Neonatal Pneumonia (Beta-Hemolytic Strep - or “GBS”)

A

This is the most common type of pneumonia in newborns. It’s acquired during exit of the dirty
birth canal. Premature infants arc at greater risk relative to term infants. It has some different
looks when compared to other pneumonias (why I discuss it separately).

How well did you know this?
1
Not at all
2
3
4
5
Perfectly
153
Q

Neonatal Pneumonia (Beta-Hemolytic Strep - or “GBS”)

trivia

A
  • It often has low lung volumes (other pneumonias have high)
  • Granular Opacities is a buzzword (for this and SDD)
  • Often (25%) has pleural effusion (SDD will not)
  • LESS likely to have pleural effusion compared to the non Beta hemolytic version (25% vs 75%)
How well did you know this?
1
Not at all
2
3
4
5
Perfectly
154
Q

Neonatal Pneumonia (not Beta-Hemolytic Strep - “Non GB” or “Non GBS”)

A

Lots of causes. Typical look is patchy, asymmetric pcrihilar densities, effusions, and
hyperinflation. Will look similar to surfactant deficient disease but will be full term. Effusions are
also much more likely (they arc rare in SDD).

How well did you know this?
1
Not at all
2
3
4
5
Perfectly
155
Q

Persistent Pulmonary HTN

A

Also called “persistent fetal circulation”. Normally, the high pulmonary pressures seen in utero
(that cause blood to shunt around the lungs) decrease as soon as the baby takes his/her first breath.
Dr. Goljan (Step 1 wizard) calls this a “miracle,” and used this basic physiology to deny
evolution. When high pressures persist in the lungs it can be primary (the work of Satan), or
secondary from hypoxia (meconium aspiration, pneumonia, etc…). The CXR is going to show
the cause of the pulmonary HTN (pneumonia), rather than the HTN itself.

How well did you know this?
1
Not at all
2
3
4
5
Perfectly
156
Q

When I say “Post Term Baby, ”

A

You Say Meconium Aspiration

How well did you know this?
1
Not at all
2
3
4
5
Perfectly
157
Q

When I say “C-Section, ”

A

You say Transient Tachypnea

How well did you know this?
1
Not at all
2
3
4
5
Perfectly
158
Q

When I say “Maternal Sedation ”

A

You say Transient Tachypnea

How well did you know this?
1
Not at all
2
3
4
5
Perfectly
159
Q

When I say “Premature”

A

You say RDS

How well did you know this?
1
Not at all
2
3
4
5
Perfectly
160
Q

Solving Cases Using lung Volumes

High (flat diaphragms)

A
  • Meconium Aspiration
  • Transient Tachypnea
  • Non BH Neonatal Pneumonia
How well did you know this?
1
Not at all
2
3
4
5
Perfectly
161
Q

Solving Cases Using lung Volumes

low

A
  • Surfactant Deficiency (no pleural effusion)

* Beta-Hemolytic Pneumonia (gets pleural effusions)

How well did you know this?
1
Not at all
2
3
4
5
Perfectly
162
Q

Pulmonary Interstitial Emphysema (PIE)

A

When you have surfactant deficiency and they put you on a ventilator (which pulverizes your
lungs with PEEP), you can end up with air escaping the alveoli and ending up in the
interstitium and lymphatics. On CXR it looks like linear lucencies (buzzword). It’s a
warning sign for impending Pneumothorax. Most cases o f PIE occur in the first week of time
(bronchopulmonary dysplasia - which looks similar - occurs in patients older than 2-3
weeks). Surfactant therapy can also mimic PIE. The treatment is to switch ventilation
methods and/or place them PIE side down.

How well did you know this?
1
Not at all
2
3
4
5
Perfectly
163
Q

Pulmonary Interstitial Emphysema (PIE)

trivia

A
  • Consequence of ventilation
  • Usually occurs in the first week of life
  • Buzzword = Linear Lucencies
  • Warning Sign for Impending Pneumothorax
  • Treatment is to put the affected side down
How well did you know this?
1
Not at all
2
3
4
5
Perfectly
164
Q

Pulmonary Interstitial Emphysema (PIE)

zebra

A

A total zebra is the progression of PIE to a large cystic mass. The thing can even cause
mediastinal mass effect.

How well did you know this?
1
Not at all
2
3
4
5
Perfectly
165
Q

Chronic Lung Disease - CLD / (Bronchopulmonary Dysplasia - BPD)

A

This is the kid bom premature (with resulting surfactant deficiency), who ends up being
tortured in ventilator purgatory. His/her tiny little lungs take a ferocious ass whipping from
positive pressure ventilation and oxygen toxicity — “barotrauma” they call it. This beating
essentially turns the lungs into scar, inhibiting their ability to grow correctly. That is why
people call this “a disease of lung growth impairment.”

How well did you know this?
1
Not at all
2
3
4
5
Perfectly
166
Q

Chronic Lung Disease - CLD / (Bronchopulmonary Dysplasia - BPD)

classic vignetter

A

Prolonged ventilation in a tiny (<1000 grams), premature kid (<32 weeks)

How well did you know this?
1
Not at all
2
3
4
5
Perfectly
167
Q

Chronic Lung Disease - CLD / (Bronchopulmonary Dysplasia - BPD)

classic look

A

Alternating regions of fibrosis (coarse reticular opacities), and hyper-aeration (cystic lucencies).

How well did you know this?
1
Not at all
2
3
4
5
Perfectly
168
Q

Chronic Lung Disease - CLD / (Bronchopulmonary Dysplasia - BPD)

buzzword

A

“Band like opacities”

How well did you know this?
1
Not at all
2
3
4
5
Perfectly
169
Q

Classic This vs That: CLD vs PIE

A

PIE = First week of life,
CLD = After 3 to 4
weeks’ postnatal age

How well did you know this?
1
Not at all
2
3
4
5
Perfectly
170
Q

Pulmonary Hypoplasia

A

This can be primary or secondary. Secondary causes seem to lend themselves more readily to multiple
choice questions. Secondary causes can be from decreased hemi-thoracic volume, decreased vascular
supply, or decreased fluid. The most common is the decreased thoracic volume, typically from a space
occupying mass such as a congenital diaphragmatic hernia (with bowel in the chest), but sometimes
from a neuroblastoma or sequestration. Decreased fluid, refers to the Potter Sequence (no kidneys ->
no pee -> no fluid -> hypoplastic lungs).

How well did you know this?
1
Not at all
2
3
4
5
Perfectly
171
Q

Bronchopulmonary Sequestration

A

These are grouped into intralobar and extralobar with the distinction being which has a pleural
covering. The venous drainage is different (intra to pulmonary veins, extra to systemic veins). You
can NOT tell the difference radiographically. The practical difference is age of presentation;
intralobar presents in adolescence or adulthood with recurrent pneumonias, extralobar presents in
infancy with respiratory compromise.

How well did you know this?
1
Not at all
2
3
4
5
Perfectly
172
Q

Bronchopulmonary Sequestration

intralobar overview

A

Much more common (75%). Presents
in adolescence or adulthood as recurrent pneumonias
(bacteria migrates in from pores of Kohn). Most
commonly in the left lower lobe posterior segment
(2/3s). Uncommon in the upper lobes. In
contradistinction from extralobar sequestration, it is
rarely associated with other developmental
abnormalities. Pathology books love to say “NO
pleural cover ” - but you can’t see that shit on CT or MR.

How well did you know this?
1
Not at all
2
3
4
5
Perfectly
173
Q

Bronchopulmonary Sequestration

intralobar quick

A

more common

presents in adolescence

recurrent infections

no pleural cover

pulmonary venous drainage

How well did you know this?
1
Not at all
2
3
4
5
Perfectly
174
Q

Bronchopulmonary Sequestration

extralobar overview

A

Less common of the two (25%).
Presents in infancy with respiratory compromise
(primarily because of the associated anomalies -
Congenital cystic adenomatoid malformation
(CCAM), congenital diaphragmatic hernia, vertebral
anomalies, congenital heart disease, pulmonary
hypoplasia). It rarely gets infected since it has its own
pleural covering. These are sometimes described as
part of a bronchopulmonary foregut malformation, and
may actually have (rarely) a patent channel to the
stomach, or distal esophagus. Pathology books love to
say “has a pleural cover ” - but you can’t see that shit
on CT or MR.

How well did you know this?
1
Not at all
2
3
4
5
Perfectly
175
Q

Bronchopulmonary Sequestration

extralobar quick

A

less common

presents in infancy

associated congenital anomalies

How well did you know this?
1
Not at all
2
3
4
5
Perfectly
176
Q

Bronchopulmonary Sequestration

gamesmanship

A

I say recurrent pneumonia in same area, you say intralobar sequestration.

How well did you know this?
1
Not at all
2
3
4
5
Perfectly
177
Q

Bronchogenic Cysts

A

Typically an incidental finding. They are generally solitary and unilocular. They typically do NOT communicate with the airway, so if they have gas in them you should worry about infection.

How well did you know this?
1
Not at all
2
3
4
5
Perfectly
178
Q

Congenital Cystic Adenomatoid Malformation (CCAM)

A

As the name suggests it’s a malformation of adenomatoid stuff that replaces normal lung. Most
of the time it only affects one lobe. There is no lobar preference (unlike CLE which favors the
left upper lobe). There are cystic and solid types (type 1 cystic, type 3 solid, type 2 in the
middle). There is a crop of knuckle heads who want to call these things CPAMs and have 5
types, which I ’m sure is evidence based and will really make an impact in the way these things
are treated. CCAMs communicate with the airway, and therefore at least components of them
can fill with air. Most of these things (like 90%) will spontaneously decrease in size in the third
trimester. The treatment (at least in the USA) is to cut these things out, because of the iddy bitty
theoretical risk of malignant transformation (pleuropulmonary blastoma, rhabdomyosarcoma).

How well did you know this?
1
Not at all
2
3
4
5
Perfectly
179
Q

What i f you see a systemic arterial feeder (one coming o ff the aorta) going to the CCAM ?

A

Then it’s not a CCAM, it’s a Sequestration. — mumble to yourself “nice try assholes”

How well did you know this?
1
Not at all
2
3
4
5
Perfectly
180
Q

Congenital Lobar Emphysema (CLE)

A

The idea behind this one is that you have bronchial pathology (maybe atresia depending on what
you read), that leads to a ball-valve anomaly and progressive air trapping. On CXR, it looks
like a lucent, hyper-expanded lobe.

How well did you know this?
1
Not at all
2
3
4
5
Perfectly
181
Q

Congenital Lobar Emphysema (CLE)

trivia

A
  • It’s not actually emphysema - just air trapping secondary to bronchial anomaly
  • It prefers the left upper lobe (40%)
  • Treatment is lobectomy
How well did you know this?
1
Not at all
2
3
4
5
Perfectly
182
Q

Congenital Lobar Emphysema (CLE)

gamesmanship

A

•The classic way this is shown in case conference or case books is with a series of CXRs.
The first one has an opacity in the lung (the affected lung clears fluid slower than normal
lung). The next x-ray will show the opacity resolved. The following x-ray will show it
getting more and more lucent. Until it’s actually pushing the heart over.

How well did you know this?
1
Not at all
2
3
4
5
Perfectly
183
Q

Congenital Diaphragmatic Hernia (CDHs)

A

Most commonly they are Bochdalek type. B is in the Back - they are typically posterior and to
the left. The appearance on CXR is usually pretty obvious.

How well did you know this?
1
Not at all
2
3
4
5
Perfectly
184
Q

Congenital Diaphragmatic Hernia (CDHs)

trivia

A
  • Usually in the Back , and on the left (Bochdalek)
  • If it’s on the right - there is an association with GBS Pneumonia
  • Mortality Rate is related to the degree o f pulmonary Hypoplasia
  • Most have Congenital Heart Disease
  • Essentially all are malrotated
How well did you know this?
1
Not at all
2
3
4
5
Perfectly
185
Q

Congenital Diaphragmatic Hernia (CDHs)

gamesmanship

A

•One trick is to show the NG tube curving into the chest.

How well did you know this?
1
Not at all
2
3
4
5
Perfectly
186
Q

Locational Strategy

Left Upper lobe

A
Think Congenital Lobar
Emphysema (CLE) first
But, remember CCAM has no
lobar prevalence, so it can be
anywhere
How well did you know this?
1
Not at all
2
3
4
5
Perfectly
187
Q

Locational Strategy

left lower lobe

A

Think Sequestration First
Congenital Diaphragmatic
Hernia (CDHs) favors this
side too

How well did you know this?
1
Not at all
2
3
4
5
Perfectly
188
Q

Special Situations in Peds Chests

viral

A

In all ages this is way more common than bacterial infection. Peribronchial edema is the
buzzword for the CXR finding. “Dirty” or “Busy” Hilum. You also end up with debris and mucus in
the airway which causes two things (1) hyperinflation and (2) subsegmental atelectasis. Respiratory
Syncytial Virus (RSV) - This will cause the typical non-specific viral pattern as well. However,
there is the classic testable predilection to cause a segmental or lobar atelectasis — particularly in the
right upper lobe.

How well did you know this?
1
Not at all
2
3
4
5
Perfectly
189
Q

Special Situations in Peds Chests

Round Pneumonia

A

Kids get round pneumonia. They love to show
this, and try to trick you into thinking it’s a mass. Younger than 8 you are
thinking round pneumonia, round pneumonia, round pneumonia - with S.
Pneumonia being the culprit. The PhD trivia is that these occur because you
don’t have good collateral ventilation pathways. Round pneumonia is
usually solitary, and likes the posterior lower lobes. Take home message: No
CT to exclude cancer, just get a follow up x-ray.

How well did you know this?
1
Not at all
2
3
4
5
Perfectly
190
Q

Special Situations in Peds Chests

Neonatal atypical peripheral atelectasis (NAPA)

A

It is best to think about this as a cousin or uncle of Round Pneumonia (they are in the
same family). It is essentially the same thing except it is peripheral. The
classic look is a round, pleural based “mass” in the apex of the lung.
Similarly to the “round pneumonia” this is a transient finding and will
resolve as the primary process improves.

How well did you know this?
1
Not at all
2
3
4
5
Perfectly
191
Q

Special Situations in Peds Chests

Lipoid Pneumonia

A

Classic history is a parent giving their newborn a teaspoonful of olive oil
daily to cultivate “a spirit o f bravado and manliness. ” Although this seems like a pretty solid plan, and
I can’t fault their intentions - it’s more likely to result in chronic fat aspiration. Hot Sauce is probably a
better option. Most people will tell you that bronchoalveolar lavage is considered the diagnostic
method of choice. CXR nonspecific - it is just airspace opacities. CT is much more likely to be the
modality used on the exam. The classic finding is low attenuation (-30 to -100 HU) within the
consolidated areas reflecting fat content

How well did you know this?
1
Not at all
2
3
4
5
Perfectly
192
Q

Special Situations in Peds Chests

Bronchial Foreign Body

A

The key concept is that it causes air trapping.
The lung may look more lucent (from air
trapping) on the affected side. You put the
affected side down and it will remain lucent
(from air trapping). Another random piece of
trivia is that under fluoro the mediastinum will
shift AWAY from the affected side on
expiration.

How well did you know this?
1
Not at all
2
3
4
5
Perfectly
193
Q

Special Situations in Peds Chests

Swyer James

A

This is the classic unilateral lucent lung. It typically occurs after a viral lung
infection in childhood resulting in post infectious obliterative bronchiolitis. The size of the affected
lobe is smaller than a normal lobe (it’s not hyper-expanded).

How well did you know this?
1
Not at all
2
3
4
5
Perfectly
194
Q

Special Situations in Peds Chests

Papillomatosis

A

Perinatal HPV can cause these soft tissue masses within the airway and lungs.
It’s also seen in adults who smoke. “Multiple lung nodules which demonstrate cavitation” is the
classic scenario. Some testable trivia includes the 2% risk of squamous cell cancer, and that
manipulation can lead to dissemination. The appearance of cysts and nodules can look like LCH
(discussed more in the thoracic chapter), although the trachea is also involved.

How well did you know this?
1
Not at all
2
3
4
5
Perfectly
195
Q

Special Situations in Peds Chests

Sickle Cell I Acute Chest

A

Kids with sickle cell can get “Acute chest.” Acute chest actually
occurs more in kids than adults (usually between age 2-4). This is the leading cause of death in sickle
cell patients. Some people think the pathology is as such: you infarct a rib -> that hurts a lot, so you
don’t breath deep -> atelectasis and infection. Others think you get pulmonary microvascular
occlusion and infarction. Regardless, if you see opacities in the CXR of a kid with sickle cell, you
should think of this.

How well did you know this?
1
Not at all
2
3
4
5
Perfectly
196
Q

Special Situations in Peds Chests

Cystic Fibrosis

A

So the sodium pump doesn’t work and

they end up with thick secretions and poor pulmonary clearance. The real damage is done by recurrent infections.

How well did you know this?
1
Not at all
2
3
4
5
Perfectly
197
Q

Special Situations in Peds Chests

Primary Ciliary Dyskinesia

A

The motile part of
the cilia doesn’t work. They can’t clear their lungs and get
recurrent infections. These guys have lots of bronchiectasis
just like CF. BUT, this time it’s lower lobe predominant
(CF was upper lobe

How well did you know this?
1
Not at all
2
3
4
5
Perfectly
198
Q

Bronchial foreign body

key point

A

A normal inspiratory CXR is meaningless. Don’t forget that the crayon / green bean is going
to be radiolucent. You need expiratory films to elicit air trapping. Normally, the bottom lung is gonna
turn white (move less air). If there is air trapping the bottom lung will stay black.

How well did you know this?
1
Not at all
2
3
4
5
Perfectly
199
Q

Sickle Cell I Acute Chest

gamesmanship

A

(how do you know it’s sickle cell?)
•Kid with Big Heart
•Kid with bone infarcts (look at the humeral heads)
•Kid with H shaped vertebra (look on lateral)

How well did you know this?
1
Not at all
2
3
4
5
Perfectly
200
Q

Cystic Fibrosis

things to know

A

•Bronchiectasis (begins cylindrical and progresses to
varicoid)
•It has an apical predominance (lower lobes are less
affected)
•Hyperinflation
•They get Pulmonary Arterial Hypertension
•Mucus plugging (finger in glove sign)
•Men are infertile (vas deferens is missing)

How well did you know this?
1
Not at all
2
3
4
5
Perfectly
201
Q

Cystic Fibrosis

trickery

A

• Fatty Replaced Pancreas on CT
• Abdominal Films with
Constipation

Biliary Cirrhosis (from blockage
of intrahepatic bile ducts), and
resulting portal HTN
How well did you know this?
1
Not at all
2
3
4
5
Perfectly
202
Q

Primary Ciliary Dyskinesia

Things to know

A

•Bronchiectasis (lower lobes)
•50% will have Kartageners (situs inversus). So, 50%
will not
•Men are infertile (sperm tails don’t work)
•Women are sub-fertile (cilia needed to push eggs
around)

How well did you know this?
1
Not at all
2
3
4
5
Perfectly
203
Q

This vs that

cystic fibrosis vs primary ciliary dyskinesia

A
CF: Upper Lobe
Predominant -
Brochiectasis
Infertile - Men are
Missing the Vas
Deferens
Primary Ciliary dyskinesia:
Lower Lobe
Predominant -
Bronchiectasis
Infertile - Men’s
Sperm Don’t Swim
For Shit
How well did you know this?
1
Not at all
2
3
4
5
Perfectly
204
Q

Pleuropulmonary Blastoma (PPB)

A

This is a primary intrathroacic malignancy. They can look a lot like CCAMs and even have different types (cystic, mixed, solid). These
things are usually right sided, pleural based, and without chest wall invasion or calcifications.
No rib invasion (helps distinguish it from the Askin / Ewing Sarcoma o f the Chest Wall - if
they won’t tell you the age), No calcification. The more solid types can have mets to the brain
and bones. The cystic type seem to occur more in kids less than a year old, and be more benign.

How well did you know this?
1
Not at all
2
3
4
5
Perfectly
205
Q

Pleuropulmonary Blastoma (PPB)

things to know

A
  • Big Fucking Mass (B.F.M.) in the chest o f a 1-2 year old
  • Shouldn’t have an eaten-up rib (Askin tumors often do)
  • 10% o f the time they have a multilocular cystic nephroma.
How well did you know this?
1
Not at all
2
3
4
5
Perfectly
206
Q

Umbilical Venous Catheter (UVC)

A

A UVC passes from the umbilical vein to the left
portal vein to the ductus venosus to a hepatic vein to
the 1VC. You don’t want the thing to lodge in the
portal vein because you can infarct the liver. The
ideal spot is at the 1VC - Right Atrium junction.

How well did you know this?
1
Not at all
2
3
4
5
Perfectly
207
Q

Development of a “Cystic Liver Mass” (Hematoma) can suggest

A

UVC erosion into the liver.

How well did you know this?
1
Not at all
2
3
4
5
Perfectly
208
Q

Umbilical Artery Catheter (UAC)

A

A UAC passes from the umbilicus, down to the
umbilical artery, into an iliac artery then to the aorta.
Positioning counts, as the major risk factor is renal
arterial thrombosis. You want to avoid the renal
arteries by going high (T8-T10), or low (L3-L5)

How well did you know this?
1
Not at all
2
3
4
5
Perfectly
209
Q

Things to know about UACs

A
  • It goes down first
  • It should be placed either high (T8-T10) or low (L3-L5)
  • Omphalocele is a contraindication
How well did you know this?
1
Not at all
2
3
4
5
Perfectly
210
Q

UAC =

A

down then up

How well did you know this?
1
Not at all
2
3
4
5
Perfectly
211
Q

UVC =

A

straight up

How well did you know this?
1
Not at all
2
3
4
5
Perfectly
212
Q

ECMO - Extracorporeal Membrane Oxygenation

A

Neonatologists primarily use this device to torture sick babies - hopefully into revealing the
various government secrets they have stolen, or the location of their organization’s
underground lair. “Enhanced interrogation” or “temporizing measure o f last resort,” they call
it - to get around the Geneva Conventions.

How well did you know this?
1
Not at all
2
3
4
5
Perfectly
213
Q

Oh you want to pretend you can’t talk? Get the ECMO catheters!

A

Alternatively, it can be used as a last resort in neonatal sepsis, severe SSD, and meconium
aspiration. Actually, in cases of meconium aspiration ECMO actually does work (sometimes).

How well did you know this?
1
Not at all
2
3
4
5
Perfectly
214
Q

ECMO - Extracorporeal Membrane Oxygenation

types overview

A

There are two main types: (1) Veno-Arterial “V-A”,
and (2) Veno-Venous “V-V”
In both cases deoxygenated blood is removed from the
right atrium and pumped into a box (artificial lung) to
get infused with oxygen. The difference is in how it is
returned.

How well did you know this?
1
Not at all
2
3
4
5
Perfectly
215
Q

ECMO - Extracorporeal Membrane Oxygenation

V-A

A

In V-A, blood goes back to the Aorta (you can see why
this would help rest the left ventricle). The catheter is
usually placed at the origin o f the innominate or
“overlying the arch.”

How well did you know this?
1
Not at all
2
3
4
5
Perfectly
216
Q

ECMO - Extracorporeal Membrane Oxygenation

V-V

A

In V-V blood goes right back into the right atrium. In
this situation even if the lungs were totally clogged
with shit (meconium aspiration) and no oxygen
exchange was happening it wouldn’t matter because
the blood that is being pumped (RA -> RV ->
Pulmonary Artery) already has oxygen in it.

How well did you know this?
1
Not at all
2
3
4
5
Perfectly
217
Q

ECMO - Extracorporeal Membrane Oxygenation

V-A quick

A
Catheter
Position:
RA + Aorta (near
the origin of
the innominate
artery)
Catheter
Position:
RA + Aorta (near
the origin of
the innominate
artery)
How well did you know this?
1
Not at all
2
3
4
5
Perfectly
218
Q

ECMO - Extracorporeal Membrane Oxygenation

V-V quick

A

Catheter
Position:
RA+ RA
“Dual Lumen”

Lung Support

How well did you know this?
1
Not at all
2
3
4
5
Perfectly
219
Q

ECHO - Extracorporeal Membrane Oxygenation - Continued

Lung White Out = Normal

A

Mechanism is variable depending on who you asked. The way
1 understand it is that the airway pressure suddenly drops off causing atelectasis, plus you
have a change in the circulation pattern that now mimics the fetal physiology (mom =
artificial lung). Resulting oxygen tension changes lead to an edema like pattern. A multiple
choice trick could be to try and make you say it is worsening airspace disease, or reflects the
severity of the lung injury. Don’t fall for that. It’s an expected finding.

How well did you know this?
1
Not at all
2
3
4
5
Perfectly
220
Q

ECHO - Extracorporeal Membrane Oxygenation - Continued

Consequences of V-A

A

I mentioned on the prior page that they typically ligate the carotid
when they place the Arterial catheter. No surprise that they will be at increased risk for
neurologic ischemic complications as a result.

How well did you know this?
1
Not at all
2
3
4
5
Perfectly
221
Q

ECHO - Extracorporeal Membrane Oxygenation - Continued

Crucial Complication = Hemorrhage

A

The combination of anticoagulation (necessary in
ECMO) and being sick as stink puts these kids at super high risk for head bleeds. This is
why they will get screened for germinal matrix hemorrhage prior to being placed on ECMO
and then routinely screened with head ultrasounds (expertly read by the second year resident
on call).

How well did you know this?
1
Not at all
2
3
4
5
Perfectly
222
Q

Catheter Position Gamesmanship:

A

I think there are two likely ways a multiple choice question could be structured related to
ECMO catheter position. The first would be to show you a series of daily radiographs with the
latest one demonstrating migration of one or both catheters. Thats the easy way to do it.
The sneaky way would be to show you the
catheter with the lucent distal end and the dot
marker on the tip. This would be particularly
evil as there are tons of different catheter
brands and looks, but if I was going to try and
trick you, this would be the way that I would do
it.
In this case, the venous catheter looks falsely
high if judged by the lucent tip, but that round
metallic marker (near the black arrow) shows
the tip in the RA.
The arterial catheter “overlies the region of the
aortic arch,” which is normal.

How well did you know this?
1
Not at all
2
3
4
5
Perfectly
223
Q

mediastinal masses

anterior

A

Normal Thymus

Thymic Rebound

Lymphoma:

Germ Cell Tumor (GCT):

How well did you know this?
1
Not at all
2
3
4
5
Perfectly
224
Q

Normal Thymus

A

This is the most common mediastinal
“mass.” It’s terribly embarrassing to call a normal thymus a
mass, but it can actually be tricky sometimes. It can be
pretty big in kids less than 5 (especially in infants).
Triangular shape of the thymus is sometimes called the “sail
sign.” Not to be confused with the other 20 sail signs in
various parts of the body, or the spinnaker sail sign, which is
when pneumomediastinum lifts up the thymus.

How well did you know this?
1
Not at all
2
3
4
5
Perfectly
225
Q

Thymic Rebound

A

In times of acute stress (pneumonia, radiation, chemotherapy, bums), the
thymus will shrink. In the recovery phase it will rebound back to normal, and sometimes larger
than before. During this rebound it can be PET avid

How well did you know this?
1
Not at all
2
3
4
5
Perfectly
226
Q

Lymphoma:

A

This is the most common abnormal mediastinal mass in children (older
children and teenagers). Lymphoma vs Thymus can be tricky. Thymus is more in kids under 10,
Lymphoma is seen more in kids over 10. When you get around age 10, you need to look for
cervical lymph nodes to make you think lymphoma. If you see calcification, and the lesion has
NOT been treated you may be dealing with a teratoma. Calcification is uncommon in an untreated lymphoma.

How well did you know this?
1
Not at all
2
3
4
5
Perfectly
227
Q

Lymphoma:

complications

A

Compression o f SVC, Compression o f Pulmonary> Veins, Pericardial Effusion, Airway Compression.

How well did you know this?
1
Not at all
2
3
4
5
Perfectly
228
Q

Lymphoma:

all/leukemia

A

can appear very similar to Lymphoma (soft tissue mass in the anterior
mediastinum). In this scenario, most people will tell you that Lymphoma can NOT be
differentiated from Leukemia on imaging alone.

How well did you know this?
1
Not at all
2
3
4
5
Perfectly
229
Q

Germ Cell Tumor (GCT):

A

On imaging, this is a large anterior mediastinal mass arising from or at least next to the thymus. It comes in three main flavors,

How well did you know this?
1
Not at all
2
3
4
5
Perfectly
230
Q

Germ Cell Tumor (GCT):

teratoma

A

Mostly Cystic, with fat and calcium

How well did you know this?
1
Not at all
2
3
4
5
Perfectly
231
Q

Germ Cell Tumor (GCT):

seminoma

A

Bulky and Lobulated. “Straddles the midline ”

How well did you know this?
1
Not at all
2
3
4
5
Perfectly
232
Q

Germ Cell Tumor (GCT):

NSGCT

A

Big and Ugly - Hemorrhage

and Necrosis. Can get crazy and invade the lung.

How well did you know this?
1
Not at all
2
3
4
5
Perfectly
233
Q

Things that make you think the

thymus is a cancer!

A
• Abnormal Size for patients Age
(really big in a 15 year old)
• Heterogenous appearance
• Calcification
• Compression of airway or
vascular structure
How well did you know this?
1
Not at all
2
3
4
5
Perfectly
234
Q

Middle mediastinal masses

A

lymphadenopathy

duplications cysts

How well did you know this?
1
Not at all
2
3
4
5
Perfectly
235
Q

lymphadenopathy

A
Middle mediastinal lymphadenopathy is most often from
granulomatous disease (TB or Fungal), or from lymphoma.
How well did you know this?
1
Not at all
2
3
4
5
Perfectly
236
Q

Mediastinal Adenopathy Trivia

mononucleosis

A

Cause: EBV (-90%), CMV (-10%)
Classic Scenario: Adolescent with hilar adenopathy, splenomegaly,
and fatigue.
Trivia: Rash after Antibiotics (Amoxicillin)
Trivia: “No sports for 3 weeks” - to avoid splenic injury

How well did you know this?
1
Not at all
2
3
4
5
Perfectly
237
Q

Mediastinal Adenopathy Trivia

primary TB

A

Distribution of nodes in usually unilateral, right hilar, right paratracheal
Lower and Middle lobe consolidation is common

How well did you know this?
1
Not at all
2
3
4
5
Perfectly
238
Q

Mediastinal Adenopathy Trivia

histoplasmosis

A

Midwest and Southeast United States
Most have normal CXRs
Can have hilar adenopathy

How well did you know this?
1
Not at all
2
3
4
5
Perfectly
239
Q

Mediastinal Adenopathy Trivia

coccidioidomycosis

A

Southwest United States
Usually looks like consolidation and nodules
Can have hilar adenopathy

How well did you know this?
1
Not at all
2
3
4
5
Perfectly
240
Q

Mediastinal Adenopathy Trivia

lymphoma

A
Often cited as the “most common” anterior mediastinal mass in a kid.
Hilar involvement (usually bilateral) is more common with Hodgkin
How well did you know this?
1
Not at all
2
3
4
5
Perfectly
241
Q

Mediastinal Adenopathy Trivia

sarcoid

A

Uncommon in children — it usually presents in the early-mid 20s.
Case reports of early onset Sarcoid exist.

How well did you know this?
1
Not at all
2
3
4
5
Perfectly
242
Q

Duplications cysts

A

These fall into three categories (a) bronchogenic, (b) enteric,
(c) neuroenteric. The neuroenterics are traditionally posterior mediastinal.

How well did you know this?
1
Not at all
2
3
4
5
Perfectly
243
Q

Duplications cysts

bronchogenic

A

water attenuation - close to the trachea or bronchus. Trivia:
• Tend to be middle mediastinal (70%), the rest are in the hilum
• Typically filled with mucus or fluid

How well did you know this?
1
Not at all
2
3
4
5
Perfectly
244
Q

Duplications cysts

enteric/esophageal

A

water attenuation close to the esophagus (lower in the
mediastinum). Trivia:
• Abutment o f the esophagus is the key finding.
• Can communicate with the lumen of the esophagus - and have air/fluid levels.
• Usually on the right, involving the distal esophagus
• Can be middle or posterior mediastinal in location
• Second most common G1 lumen duplication cyst (distal ileum #1)

How well did you know this?
1
Not at all
2
3
4
5
Perfectly
245
Q

Posterior mediastinal masses

A

Neuroblastoma

Ewing Sarcoma

Askin Tumor (Primitive Neuroectodermal tumor of the chest wall):

Neuroenteric Cyst

Extramedullary Hematopoiesis

How well did you know this?
1
Not at all
2
3
4
5
Perfectly
246
Q

Neuroblastoma

A

This is the most common posterior mediastinal mass in a child under 2. This
is discussed in complete detail in the GU PEDs section. I’ll just mention that compared to abdominal
neuroblastoma, thoracic neuroblastoma has a better outcome. It may involve the ribs and vertebral
bodies. Also, remember that Wilms usually mets (more than neuroblastoma) to the lungs, so if it’s in
the lungs don’t forget about Wilms.

How well did you know this?
1
Not at all
2
3
4
5
Perfectly
247
Q

Neuroblastic mediasetinal tumors

most aggressive and immature to least aggressive and mature

A

neuroblastoma > ganglioneruoblastoma > ganglioneuroma

How well did you know this?
1
Not at all
2
3
4
5
Perfectly
248
Q

When Compared with Neuroblastoma, Ganglioneuromas are

A

• Less Aggressive
• More Circumscribed (less invasive)
• Less Likely to have Calcifications (although they still can)
• Pound in Older Children
These tumors can NOT be differentiated with imaging alone
(but that doesn’t mean someone can’t write a multiple choice
question asking you to try).

How well did you know this?
1
Not at all
2
3
4
5
Perfectly
249
Q

Askin Tumor (Primitive Neuroectodermal tumor of the chest wall):

A

This is now considered part of the Ewing Sarcoma spectrum, and is sometimes called an Ewing
sarcoma of the chest wall. They tend to displace adjacent structures rather than invade early on (when
they get big they can invade). They look heterogenous, and the solid parts will enhance.

How well did you know this?
1
Not at all
2
3
4
5
Perfectly
250
Q

Neuroenteric Cyst

A

By convention these are associated with vertebral anomalies (think
scoliosis, hemivertebrae, butterfly vertebrae, split cord, etc..) - think cyst protruding out of an unsealed
canal / defect. The cyst does NOT communicate with CSF, is well demarcated, and is water density.
Favor the lower cervical and thoracic regions.

How well did you know this?
1
Not at all
2
3
4
5
Perfectly
251
Q

Extramedullary Hematopoiesis

A

This occurs in patients with myeloproliferative
disorders or bone marrow infdtration (including sickle cell). Usually, this manifests as a big liver and
big spleen. However, in a minority of cases you can get soft tissue density around the spine
(paraspinal masses), which are bilateral, smooth, and sharply delineated.

How well did you know this?
1
Not at all
2
3
4
5
Perfectly
252
Q

Extramedullary Hematopoiesis

A

This occurs in patients with myeloproliferative
disorders or bone marrow infdtration (including sickle cell). Usually, this manifests as a big liver and
big spleen. However, in a minority of cases you can get soft tissue density around the spine
(paraspinal masses), which are bilateral, smooth, and sharply delineated.

How well did you know this?
1
Not at all
2
3
4
5
Perfectly
253
Q

Strategy - The Anterior Mediastinal Mass

Lymphoma

A

In a kid just assume it’s Hodgkins (which means it’s gonna involve the
thymus). Why assume Hodgkins ? Hodgkins is 4x more common than NHL. Hodgkins
involves the thymus 90% of the time.

How well did you know this?
1
Not at all
2
3
4
5
Perfectly
254
Q

Strategy - The Anterior Mediastinal Mass

How the hell do you tell a big ass normal thymus in a little baby vs a lymphoma?

A

My main move is to go age. Under 10 = Thymus, Over 10 = Lymphoma.

How well did you know this?
1
Not at all
2
3
4
5
Perfectly
255
Q

Strategy - The Anterior Mediastinal Mass

Thymic Rebound

A

If the test writer is headed in this direction they MUST either (a)
bias you with a history saying stuff like “got off chemo” or “got off corticosteroids” or
(b) show you a series of axial CTs with the thing growing and maintaining normal
morphology. I think “a” is much more likely.

How well did you know this?
1
Not at all
2
3
4
5
Perfectly
256
Q

Strategy - The Anterior Mediastinal Mass

The Funk

A

In general just think morphology / density:
• Soft Tissue - Kinda Homogenous = Think Lymphoma or Hyperplasia
• Fat = Germ Cell Tumor (Why God!? Why Klinefelters!?)
• Water = Congenital Stuff - Think Lymphangiomas

How well did you know this?
1
Not at all
2
3
4
5
Perfectly
257
Q

Strategy - The Posterior
Mediastinal Mass

Under 10

A

Think malignant,

Think neuroblastoma.

How well did you know this?
1
Not at all
2
3
4
5
Perfectly
258
Q

Strategy - The Posterior

Mediastinal Mass

A

Think benign.

If it’s a round mass- Think about
Ganglioneuromas & Neurofibromas

If it’s cystic (and there is scoliosis) think
Neuroenteric Cyst

If they show you coarse bone
trabeculation - with an adjacent mass (or
a history of anemia) - Think
Extramedullary Hematopoiesis

How well did you know this?
1
Not at all
2
3
4
5
Perfectly
259
Q

Strategy - The B.F.M.

A
“Big Fucking Mass”
If you see a B.F.M. in the chest o f a kid,
you basically have two choices:
(1) Askin Tumor (PNET / Ewings) -
* AGE 10+, look for an eaten up rib.
(2) Pleuropulmonary Blastoma
*AGE is typically less than 2.
How well did you know this?
1
Not at all
2
3
4
5
Perfectly
260
Q

Esophageal Atresia /TE fistula:

A

This can occur in multiple subtypes, with the classic ways of showing it being a frontal CXR with an NG tube stopped in the upper neck, or a fluoro study (shown lateral) with a blind ending sac or communication with the tracheal tree. There are 5 main subtypes, only 3 (shown above) are worth knowing (being familiar with) for the purpose of the exam.

How well did you know this?
1
Not at all
2
3
4
5
Perfectly
261
Q

Esophageal Atresia /TE fistula:

Things to know

A

•Diagnosis is made with a Fluoro swallow exam
•Most Important Thing To Know are the VACTERL associations (more on this later)
• The most common subtype is the N-Type (blind ended esophagus, with distal esophagus hooked up
to trachea
• Excessive Air in the Stomach = H type (can also be with N type)
• No Air in the Stomach = Esophageal Atresia
• The presence of a right arch (4%) must be described prior to surgery (changes the approach).

How well did you know this?
1
Not at all
2
3
4
5
Perfectly
262
Q

Esophageal Atresia /TE fistula:

gamesmanship

A

Fake out for TE fistula is simple aspiration. Look for the presence or absence of laryngeal penetration to tell them apart (if shown a dynamic Fluoro swallow exam).

How well did you know this?
1
Not at all
2
3
4
5
Perfectly
263
Q

VACTERL:

A

This is extremely high yield. VACTERL is a way of remembering that certain
associations are seen more commonly when together (when you see one, look for the others).

How well did you know this?
1
Not at all
2
3
4
5
Perfectly
264
Q

VACTERL:

frequency

A
V - Vertebral Anomalies (37%)
A - Anal (imperforate anus) (63%)
C - Cardiac (77%)
TE - Tracheoesophageal Fistula
or Esophageal Atresia (40%)
R - Renal (72%)
L - Limb (radial ray) - 58%
How well did you know this?
1
Not at all
2
3
4
5
Perfectly
265
Q

VACTERL:

diagnosis

A

VACTERL association is diagnosed when 3
or more of the defined anomalies affect a
patient.
Therefore, keep investigating when 1 -2 of
these anomalies are found.
The heart and kidneys are the most
commonly affected organs in this

How well did you know this?
1
Not at all
2
3
4
5
Perfectly
266
Q

VACTERL:

trivia

A

If both limbs are involved, then both kidneys tend to be involved. If one limb is involved, then
one kidney tends to be involved

How well did you know this?
1
Not at all
2
3
4
5
Perfectly
267
Q

Esophageal Atresia

stricture

A

Around 30% of kids with a repaired esophageal atresia will end up with a focal
anastomotic stricture. Strictures can also be seen with caustic ingestion (dishwashi

How well did you know this?
1
Not at all
2
3
4
5
Perfectly
268
Q

Esophageal Foreign Bodies

A

Kids love to stick things in their mouths (noses and ears).
This can cause a lot of problems including direct compression o f the airway, perforation, or even
fistula to the trachea. Stuff stuck in the esophagus needs to be removed.

How well did you know this?
1
Not at all
2
3
4
5
Perfectly
269
Q

Esophageal Foreign Bodies

A

The esophagus is a dirty sock, it flexes to accommodate that big piece of steak you didn’t even bother to chew. The trachea is rigid, like that math teacher I had in high school (who hated music… and colors), but unlike the math teacher it has a flexible membrane in the back.

The point of me mentioning this is to help you problem solve a “where is the coin ”
type question. The esophagus will accommodate the coin so it can be turned in any direction. The trachea is rigid and will force the coin to rotate
into the posterior membrane — so it will
be skinny in the AP direction.

How well did you know this?
1
Not at all
2
3
4
5
Perfectly
270
Q

Ingested Metallic Foreign Bodies

Magnets

A

One magnet is ok. Two or more magnets is a
problem. The reason is that they can attract
each other across intestinal walls leading to
obstruction, necrosis, perforation, and a law suit.
Surgical Consult

NOT an MRI (dumbass)

How well did you know this?
1
Not at all
2
3
4
5
Perfectly
271
Q

Ingested Metallic Foreign Bodies

AA or AAA Batteries

A

Less of a problem relative to other types of
batteries, but can cause serious problems if you
need them for the DVR control because Game of
Thrones season 7 is on (spoiler it sucked).

Serial plain film exams.
Remove if they stay in
the stomach for more
than 2 days

How well did you know this?
1
Not at all
2
3
4
5
Perfectly
272
Q

Ingested Metallic Foreign Bodies

Disc Batteries

A
They look like coins, except they
have two rings. The literature is
not clear, but it appears that
modern batteries rarely leak
(leaking is bad - caustic
chemicals, heavy metals etc..).

Stuck in the esophagus
= big problem, gotta
get them out within 2 hours.

Stuck in the stomach =
problem, gotta get
them out within 4 days.

How well did you know this?
1
Not at all
2
3
4
5
Perfectly
273
Q

Ingested Metallic Foreign Bodies

Coins
(Including Pennies
minted prior to 1982)

A

Copper Pennies are relatively safe.

Make sure it is not a disc battery
(coins have one order ring, disc
battery has two).

Remove if:
Retention in the
esophagus for more
than 24 hours or
Stomach for more than
28 days.
How well did you know this?
1
Not at all
2
3
4
5
Perfectly
274
Q

Ingested Metallic Foreign Bodies

Pennies
-minted after 1982

A

Those minted after 1982 contain mostly Zinc
which when combined with stomach acid can
cause gastric ulcerations, and if absorbed in
great enough quantity can cause zinc toxicosis
(which is mainly pancreatic dysfunction /
pancreatitis). The ulcers are the more likely thing
to happen, so just remember that.

So how the hell can you tell the date of a penny
that is swallowed? Either (a) the question stem
will have to say something like - “2 year old child
playing with father’s collection of 1984 pennies” ,
or the more likely (b) showing you the penny with
characteristic radiolucent holes - from erosion.

remove rom stomach

How well did you know this?
1
Not at all
2
3
4
5
Perfectly
275
Q

Ingested Metallic Foreign Bodies

lead

A

Usually Lead Paint Dust - or Any object with lead paint.

Bad because the gastric acid leads to immediate absorption.

Remove immediately
from stomach. Distal
passable must be
confirmed.

276
Q

Ingested Metallic Foreign Bodies

shar object nos

A

Esophagus = Remove Immediately
Stomach = Remove Immediately
Post Pylorus = Follow vs Surgery — if it does perforate the small bowel it
will be at the IC valve

277
Q

Pulmonary Sling

A

• The only variant that goes between the esophagus and the trachea.
• Classic question is that this is associated with tracheal stenosis (which is actually primary
and not secondary to compression).
• High association with other cardiopulmonary and systemic anomalies: hypoplastic right lung,
horseshoe lung, TE-fistula, imperforate anus, and complete tracheal rings.
• Treatment is controversial but typically involves surgical repositioning o f the artery

278
Q

Double Aortic Arch

A

most common symptomatic ascular ring anomaly

impressions on the anterior aorta and posterior trachea

279
Q

Left Arch with Aberrant Right Subclavian Artery

A

• Most Common Aortic Arch Anomaly — not
necessarily symptomatic.
• “Dysphagia Lusoria” - fancy Latin speak (therefore
high yield) for trouble swallowing in the setting of
this variant anatomy
• “Diverticulum of Kommerell” pouch like
aneurysmal dilatation of the proximal portion o f an
aberrant right subclavian artery

280
Q

Bowel Obstruction ( in the neonate 1

High

A

Midgut Volvulus /
Malrotation

Duodenal Atresia

Duodenal Web

annular pancrease

jejunal atresia

281
Q

Bowel Obstruction ( in the neonate 1

Low

A

hirschprung disease

meconium plug syndrome

ileal atresia

meconium ileus

anal atresia/colonic atresia

282
Q

Single Bubble

A

Gastric (antral or pyloric) atresia.

283
Q

Double Bubble

A

Duodenal Atresia

284
Q

Duodenal Atresia Trivia

A
• 30% have Downs
• 40% have polyhydramnios and
are premature
• The“singlc atresia” - cannulation
error
• On multiple choice test the
“double bubble” can be shown on
3rd trimester OB ultrasound, plain
film, or on MRI.
285
Q

Triple Bubble

A

Jejunal Atresia. When you call jejunal atresia,
you often prompt search for additional atresias (colonic). Just remember
!^ ,’ P that jejunal atresia is secondary to a vascular insult during development.

286
Q

multiple atresia

A

vascular error

287
Q

Single Bubble with Distal Gas

A

Can mean nothing (lotta air swallowing). If the clinical
history is bilious vomiting , this is ominous and can
be midgut volvulus (surgical emergency). Next test
would be emergent Upper GI.

288
Q

Double Bubble with Distal Gas

A

Seeing distal gas excludes duodenal atresia. The DDx is a
duodenal web, duodenal stenosis, or midgut
volvulus. Next step would be upper GI.

289
Q

Multiple Diffusely Dilated Loops

A

Suggestive o f a low obstruction (ileum or colon).
Next step is contrast enema. If the contrast enema is
normal you need to follow with upper GI (to
exclude an atypical look for midgut volvulus).

290
Q

Mildly Dilated, Scattered Loops

A

“Sick Belly” - Can be seen with proximal or distal obstruction. Will need Upper GI and contrast enema.

291
Q

Malrotation

A

Normally, the developmental rotation o f the gut places the ligament of
Trietz to the left o f the spine (at the level o f the duodenal bulb). If mother nature fucks up and
this doesn’t happen, you end up with the duodenum to the right o f the midline (spine). These
patients are at increased risk for midgut volvulus and internal hernias. If you see the
appearance o f malrotation and the clinical history is bilious vomiting, then you must suspect
midgut volvulus

292
Q

Malrotation

trivia

A

•Associated with Heterotaxy Syndromes. Associated with Omphaloceles.
•Classically shown as the SMA to the right of the SMV (on US or CT).
•False Positive on UGI - Distal Bowel Obstruction displacing the duodenum (because of
ligamentous laxity).

293
Q

I say “Non-Bilious Vomiting”

A

You Say Hypertrophic Pyloric Stenosis

next step ultrasound

294
Q

I say “Bilious Vomiting”

A

You Say Mid Gut Volvulus (till proven otherwise)

Next Step ? Upper GI

295
Q

Corkscrew Duodenum

A

This is diagnostic of
midgut volvulus (surgical emergency). The appearance
is an Aunt Minnie.

296
Q

Ladd’s Bands

A

In older children (or even adults)
obstruction in the setting of malrotation will present as
intermittent episodes o f spontaneous duodenal
obstruction. The cause is not midgut volvulus (a
surgical emergency) but rather kinking from Ladd’s Bands.

297
Q

Ladd’s Bands

what is it

A

We are talking
about a fibrous stalk of peritoneal tissues that fixes the
cecum to the abdominal wall, and can obstruct the duodenum.

298
Q

Complete Duodenal Obstruction

A

Strongly associated with midgut volvulus. If you were thinking duodenal atresia, look for distal air (any will do) to exclude that thought.

299
Q

Ladd’s Procedure

A
Procedure to prevent midgut
volvulus. Traditionally, the
Ladd’s Bands are divided, and
the appendix is taken out. The
small bowel ends up on the
right, and the large bowel ends
up on the left. They are fixed
in place by adhesions (just by
opening the abdomen).
It is still possible to develop
volvulus post Ladd’s (but it’s
rare - 2-5%).
300
Q

Ladd’s Procedure

steps

A
  • Divide the Adhesive Ladds Bands
  • Widen the Mesentery to a safe distance
  • Take out the appendix (bill extra for that)
301
Q

Partial Duodenal Obstruction

A

If the kid is vomiting this might be from extrinsic
narrowing (Ladd band, annular pancreas), or
intrinsic (duodenal web, duodenal stenosis).
You can’t tell.

302
Q

Hypertrophic Pyloric Stenosis

A

Thickening of the gastric pyloric musculature, which results in progressive obstruction. Step 1 buzzword is “non-bilious vomiting.” Here is the most likely multiple choice trick; this does NOT occur at birth or after 3 months.

There is a specific age range o f 2-12 weeks (peak at 3-6 weeks).

Criteria is 4mm and 14mm (4mm single wall, 14mm length).

The primary differential is pylorospasm (which will relax during exam). The most common pitfall during the exam is gastric over distention, which can lead to displacement o f the antrum and pylorus - leading to false negative.

False positive can result from off axis measurement.

The phenomenon of “paradoxical aciduria” has been described, and is a common buzzword.

303
Q

Gastric Volvulus

Organoaxial

A
The greater
curvature flips over the lesser
curvature (rotation along the long
axis). This is seen in old ladies
with paraesophageal hernias.
304
Q

Gastric Volvulus

•Mesenteroaxial

A
Twisting over
the mesentery (rotation along short
axis). The antrum flips near the
GE junction. Can cause ischemia
and needs to be fixed.
Additionally this type causes
obstruction. This type is more
common in kids.
305
Q

Duodenal Web:

A

This is best thought
of as “almost duodenal atresia. ” The
reason I say that is, just like duodenal
atresia, this occurs from a failure to
canalize, but instead o f a total failure of
canalization (like duodenal atresia) this
bowel is only partially canalized, leaving
behind a potentially obstructive web.

306
Q

Duodenal Web

things to know

A
- Because the web is distal to ampulla of
Vater - you get bile-stained emesis
- Associated with malrotation and
Downs syndrome
- The “wind sock” defonnity is seen
more in older kids - where the web-like
diaphragm has gotten stretched.
307
Q

Annular Pancreas

A

Essentially an embryologic screw up (failure o f ventral bud to rotate
with the duodenum), that results in encasement o f the duodenum.
In Kids = Think Duodenal Obstruction
In Adults = Think Pancreatitis

308
Q

Annular Pancreas

on ct

A

Look for pancreatic tissue
(same enhancement as the nearby
normal pancreas) encircling the
descending duodenum.

309
Q

Annular Pancreas

on fluoro

A
Look for an extrinsic
narrowing of the duodenum. Obviously
this is non-specific (typical barium -
voodoo), use the location and clinical
history to bias yourself.
310
Q

“Low Obstruction” in a Neonate

A

You basically have 4 choices: Normal,

Short Microcolon, Long Microcolon, and a Caliber Change from micro to normal.

311
Q

Short Microcolon -

A

• Think about Colonic Atresia

312
Q

Long Microcolon

“Meconium Ileus

A

ONLY in patients with CF.
The pathology is the result of thick sticky meconium
causing obstruction o f the distal ileum. Contrast will
reach ileal loops, and demonstrate multiple filling
defects (meconium). This can be addressed with an
enema.

313
Q

Long Microcolon

Distal Ileal Atresia

A

This is the result o f
intrauterine vascular insult. Contrast will NOT reach
ileal loops. This needs surgery.

314
Q

Caliber Change

•Small Left Colon (Meconium Plug)
Syndrome

A

This is a transient functional colonic
obstruction, that is self limited and relieved by
contrast enema.
*Most Testable Fact:
It is NOT associated with CF.
*2nd Most Testable Fact:
It is seen in infants of diabetic mothers 01
if mom received magnesium sulfate for eclampsia

315
Q

Caliber Change

•Hirschsprung Disease

A

Failure of the
ganglion cells to migrate and innervate the distal
colon. Affected portions o f the colon are small
in caliber, whereas the normally innervated
colon appears dilated.

316
Q

•Hirschsprung Disease

Trivia

A
  • It’s 4:1 more common in boys.
  • 10% association with Downs.
  • Diagnosis is made by rectal biopsy.
317
Q

•Hirschsprung Disease

How it can be Shown

A

-Enema - Rectum smaller than the Sigmoid
“Recto-sigmoid ratio < 1”
-Enema - Rectum with “sawtooth pattern”
Represents bowel spasm

318
Q

•Hirschsprung Disease

Presentation:

A
(1) Newborn who fails to have BM > 48
hours (or classically > 72 hours)
(2) “Forceful passage o f meconium after
rectal exam”
(3) One month old who shows up “sick as
stink” with NEC bowel
319
Q

Total Colonic Aganglionosis

A

This is a super rare variant o f Hirschsprungs, and
can mimic microcolon. The piece of commonly asked trivia is that it can also involve the
terminal ileum.

320
Q

Meconium Peritonitis

A

This is a potential complication o f bowel atresia
or meconium ileus. It has a very characteristic
look. It’s a calcified mass in the mid
abdomen , traditionally shown on plain film. It
is the result of a sterile peritoneal reaction to an
in-utero bowel perforation.
Usually, the perforation seals off prior to birth
and there is no leak.

321
Q

Imperforate or Ectopic Anus

A

Imperforate or Ectopic Anus

322
Q

Is a y “Baby with no asshole”

A

you say “VACTERL ”

you say “Screening US fo r tethered cord”

323
Q

-Obstruction in an Older Child

ddx

A

Classic D D x “A IM ” -
Appendicitis, Adhesions
Inguinal Hernia, Intussusception
Midgut Volvulus, Meckels

324
Q

Appendic itis

A

In children older than 4 this is the most common cause for bowel obstruction. I f they
show this in the PEDs section it’s most likely to be on ultrasound. In that case you can expect a blindending
tube, non-compressible, and bigger than 6 mm.

325
Q

Inguinal He rn ia:

A

Big points are that indirect hernias
are more common in kids, they are lateral to the inferior epigastric, and incarceration is the most common
complication. Umbilical hernias are common in kids, but rarely incarcerate.
Trivia to know: This is the most common cause o f obstruction in boy 1 month - 1 year

326
Q

Intussusc eption

A

The age range is 3 months - 3 years, before or after that you should think o f lead
points (90% between 3 months and 3 years d o n ’t have lead points). The normal mechanism is forward
peristalsis resulting in invagination o f proximal bowel (the intussusceptum) into lumen o f the distal bowel
(the intussuscipiens). They have to be bigger than 2.5 cm to ma tte r (in most cases- these are enterocolic),
those that are less than 2.0 cm are usually small bowel-small bowel and may reduce spontaneously within
minutes. Just like an appendix, in the peds section, I would anticipate this shown on ultrasound as either
the target sign or pseudo-kidney.

327
Q

Intussusc eption

reducing trivia

A

exam)
• Recurrence: Usually within 72 hours
• Success Rates - 80-90% with air (Henoch-Schonlein purpura has a
reduced success rate)
• Risk o f Perforation - 0.5%
• Air causes less peritonitis (spillage o f fecal material) than barium
• Pressure should NOT exceed 120 mniHg
• Needle decompression would be the next step if they perforate and get tension pneumoperitoneum.

328
Q

Intussusc eption

ways to ask

A

(1) what is it ?-
these should be straight forward as targets or pseudo kidneys, (2) lead
points - stu ff like HSP (vasculitis), Meckle diverticulum, enteric
duplication cysts, and (3) reduction trivia.

329
Q

Me cke ls Diverticulum

A

This is a congenital diverticulum o f the distal ileum. A piece o f total trivia is
that it is a persistent piece o f the omphalomesenteric duct. Step 1 style, “rule o f 2s” occurs in 2% o f the
population, has 2 types o f heterotopic mucosa (gastric and pancreatic), located 2 feet from the IC valve, it’s
usually 2 inches long (and 2 cm in diameter), and usually has symptoms before the child is 2. I f it has
gastric mucosa (the ones that bleed typically do) it will take up Tc-Pertechnetate ju s t like the stomach
(hence the Meckel’s scan).

330
Q

Me cke ls Diverticulum

trivia

A
  • Can get diverticulitis in the Meckels (mimic appendicitis)
  • GI Bleed from Gastric Mucosa (causes 30% o f symptomatic cases)
  • Can be a lead point for intussusception (seen with inverted diverticulum)
  • Can Cause Obstruction
331
Q

Gastroschisis

A

Extra-abominal evisceration of neonatal bowel (sometimes stomach and liver)
through a paraumbilical wall defect.

332
Q

Gastroschisis

things to know

A
  • It does NOT have a surrounding membrane (omphalocele docs)
  • It’s always on the RIGHT side.
  • Associated anomalies are rare (unlike omphalocele).
  • Maternal Serum AFP will be elevated (higher than that of omphalocele)
  • Outcome is usually good
  • For some reason they get bad reflux after repair.
  • Associated with intestinal atresias.
333
Q

Omphalocele

A

Congenital midlinc defect, with herniation of gut at the base of the umbilical cord.

334
Q

Omphalocele

trivia

A

•It DOES have a surrounding membrane (gastroschisis does
not)
•Associated anomalies are common (unlike gastroschisis)
•Trisomy 18 is the most common associated chromosomal
anomaly
•Other associations: Cardiac (50%), Other GI, CNS, GU,
Turners, Klinefelters, Beckwith-Wiedemann, Pentalogy of
Cantrell
•Outcomes are not that good, because of associated
syndromes.
•Umbilical Cord Cysts (Allantoic Cysts) are associated

335
Q

Pentalogy of Cantrell

A
  1. Omphalocoele
  2. Ectopia Cordis
    (abnormal location o f heart)
  3. Diaphragmatic Defect
  4. Pericardial Defect
    or Sternal Cleft
  5. Cardiovascular malformations
336
Q

This vs T hat— Gastroschisis vs Omphalocele

Gastroschisis

A

Herniated bowel loop through the
ventral body wall

NOT surrounded by a membrane

Umbilicus is Normal - positioned to the left o f the
defect (defect is on the right)

defect is on the right

asociated anomalies are rare

associated with intestinal atresias

cause: probably environment which explains an association with bowel atresia

337
Q

This vs T hat— Gastroschisis vs Omphalocele

omphalocele

A

Herniated bowel loop through the
ventral body wall

Surrounded by a membrane (peritoneum)

Umbilicus contains herniated b ow e l, and therefore is
NOT normal

defect is midline

High A ssociation with Cardiac (50%) defects and
Chromosomal Abnormalities

Multiple Syndromic Associations including Turners,
Klinefelters, Beckwith-Wiedemann, and the
Legendary Pentalogy o f Cantrell

Cause: Probably Genetic - which explains the
associations with the various syndromes

338
Q

Duodenal Hematoma

A

Classic injury from bicycle handlebars (or child abuse!. You can also
see this as a complication from endoscopy. You could be shown retroperitoneal gas as a way to
suggest perforation.

339
Q

Enteric Duplication Cysts

A

These are developmental anomalies (failure to canalize). They
don’t have to communicate with the GI lumen but can. They are most commonly in the ileal region
(40%). They have been known to cause in utero bowel obstruction / perforation.

340
Q

Enteric Duplication Cysts

strategy

A

Strategy: A common way to show this is a cyst in the abdomen (on ultrasound). If you have a random
cyst in the abdomen you need to ask yourself - “does this have gut signature? ”
• Cyst with Gut Signature = Enteric Duplication Cyst
• Cyst without Gut Signature = Omental Cyst
• WTF is “Gut Signature ?” - It’s alternating bands of hyper and hypo echoic signal - supposedly
representing different layers of bowel.

341
Q

Enteric Duplication Cysts

trivia

A

30% of the time they are associated with vertebral anomalies.

342
Q

Distal Intestinal Obstruction Syndrome

A

This is a cause of bowel obstruction in an older
kid (20 year old) with cystic fibrosis. This is sometimes called the “meconium ileus equivalent, ”
because you end up with a distal obstruction (as the name implies) secondary to dried up thick stool. It
more commonly involves the ileum / right colon. Kids who get this, are the ones who aren’t compliant
with their pancreatic enzymes.

343
Q

Mesenteric Adenitis

A

Self-limiting, usually viral inflammatory condition of mesenteric lymph
nodes. It is a classic clinical mimic of appendicitis. The finding is a cluster of large right lower quadrant lymph nodes.

344
Q

Necrotising Enterocolitis (NEC)

A

This is bad news. The general thinking is that you have an immature bowel mucosa (from being
premature or having a heart problem), and you get translocated bugs through this immature bowel. It’s
best thought of as a combination of ischemic and infective pathology.

345
Q

Necrotising Enterocolitis (NEC)

who gets it

A
  • Premature Kids (90% within the first 10 days of life)
  • Low Birth Weight Kids ( < 1500 grams)
  • Cardiac Patients (sometimes occult) - they can be full term
  • Kids who had perinatal asphyxia
  • Hirschsprung Kids that go home and come back - they present around month 1.
346
Q

Necrotising Enterocolitis (NEC)

what does it look like

A

• Pneumatosis - most definitive finding; Look for Portal Venous Gas Next
• Focal Dilated Bowel (especially in the right lower quadrant) - the terminal ileum / right colon is the
region most affected by NEC
• Featureless small bowel , with separation (suggesting edema).
• Unchanging bowel gas pattern — this would be a dirty trick - showin

347
Q

Necrotising Enterocolitis (NEC)

Pneumatosis vs Poop - The age old question

A
  • First question - has the kid been feed? No food = No poop.

* Second question - is it staying still? Poop will move, Pneumatosis will stay still.

348
Q

Necrotising Enterocolitis (NEC)

trivia

A

Use of maternal breast milk is the only parameter associated with decreased incidence of NEC.

349
Q

Pancreas

CF

A

The pancreas is nearly always (90%) with CF patients.
Inspissated secretions cause proximal duct obstruction leading to the
two main changes in CF: (1) Fibrosis (decreased T1 and T2 signal) and
the more common one (2) fatty replacement (increased Tl).
Patients with CF diagnosed as adults tend to have more pancreas
problems than those diagnosed as children. Those with residual
pancreatic exocrine function can have bouts of recurrent acute
pancreatitis. Small (l-3mm) pancreatic cysts are common.

350
Q

Pancreas

CF

trivia

A
  • Complete fatty replacement is the most common imaging finding in adult CF
  • Enlarged with fatty replacement = lipomatous pseudohypertrophy of the pancreas.
  • Fibrosing Colonopathy: Thick walled right colon as a complication of enzyme replacement therapy
351
Q

Shwachman-Diamond Syndrome

A

The 2nd most common cause of pancreatic insufficiency in kids (CF #1). Basically, it’s a kid with diarrhea, short stature, and
eczema. Will also cause lipomatous pseudohypertrophy of the pancreas.

352
Q

Dorsal Pancreatic Agenesis

A

You only have a ventral bud (the dorsal bud forgets
to form). Since the dorsal buds makes the tail, the appearance is that of a pancreas without a
tail. All you need to know is that (1) this sets you up for diabetes (most o f your beta cells are
in the tail), and (2) it’s associated with polysplenia.

353
Q

Pancreatitis

A

The most common cause of pancreatitis in peds is trauma (seat belt).

354
Q

Pancreatitis NAT

A

Another critical point to make is that non-accidental trauma can present as pancreatitis.
If the kid isn’t old enough to ride a bike (handle bar injury) or didn’t have a car wreck (seat belt injury) you need to think NAT.

355
Q

.Tumors of the Pediatric Pancreas

A

Even at a large pediatric hospital its uncommon to see more

than 1-2 of these a year. Obviously, they are still fair game for multiple choice

356
Q

Solid and Papillary Epithelial Neoplasm (SPEN)

A

The most common pediatric solid
tumor. It’s found in female adolescents (usually asian, or black). The outcomes are pretty good after
surgical resection. If you get shown a case in the peds setting this is probably it.

357
Q

Peds Pancreatic Mass

A

Age 1 = Pancreatoblastoma
Age 6 = Adenocarcinoma
Age 15 = SPEN

358
Q

Liver masses

age 0-3

A

Infantile Hepatic Hemangioma

Hepatoblastoma

Mesenchymal Hamartoma

359
Q

Infantile Hepatic Hemangioma

A

Often 1. Associated with high output CHF, this is
classically shown as a large heart on CXR plus a mass in
the liver. The aorta above the hepatic branches o f the
celiac is often enlarged relative to the aorta below the
celiac because o f differential flow. Skin hemangiomas
are present in 50%. Endothelial growth factor is
elevated. These can be associated with Kasabach-Merritt
Syndrome (the platelet eater).

360
Q

Infantile Hepatic Hemangioma

how do they do

A

Actually well. They tend to spontaneously involute without therapy over
months-years - as they progressively calcify.

361
Q

Hepatoblastoma:

A

Most common primary liver tumor o f childhood (< 5).
The big thing to know is that it’s associated with a bunch
o f syndromes - mainly hemi-hypertrophy, Wilms,
Beckwith-Weidemann crowd. Prematurity is a risk factor.
This is usually a well circumscribed solitary right sided
mass, that may extend into the portal veins, hepatic veins,
and IVC. Calcifications are present 50% o f the time. AFP
is elevated. Another piece of trivia is the hepatoblastoma
may cause a precocious puberty from making bHCG.

362
Q

Hepatoblastoma:

I would know 3 things

A

(1) Associated with Wilms, (2) AFP, (3) Precocious Puberty

363
Q

Mesenchymal Hamartoma

A

This is the predominately cystic mass (or multiple cysts), sometimes
called a “developmental anomaly.” Because it’s a “developmental
anomaly” it shouldn’t surprise you that the AFP is negative.
Calcifications are UNCOMMON. What is common is a large portal
vein branch feeding the tumor.

364
Q

liver masses

Age > 5

A

HCC

Fibrolamellar Subtype

Undifferentiated Embryonal Sarcoma

365
Q

HCC

A
This is actually the second most common liver cancer in kids. You’ll see them in kids
with cirrhosis (biliary atresia, Fanconi syndrome, glycogen storage disease). AFP will be elevated.
366
Q

Fibrolamellar Subtype

A

This is typically seen in younger patients (<35) without
cirrhosis and a normal AFP. The buzzword is central scar. The scar is similar to the one seen
in FNH with a few differences. This scar does NOT enhance, and is T2 dark (the FNH scar is
T2 bright). As a point of trivia, this tumor is Gallium avid. This tumor calcifies more often
than conventional HCC.

367
Q

Undifferentiated Embryonal Sarcoma

A

This is the pissed off cousin of the
mesenchymal hamartoma. It’s also cystic, but the mass is much more aggressive. It will be a
hypodense mass with septations and fibrous pseudocapsule. This mass has been known to
rupture.

368
Q

liver masses any age

A

Mets: Think about Wilms tumor or Neuroblastoma
Now. there are several other entities that can occur in the liver o f young children / teenagers
including; Hepatic Adenoma, Hemangiomas, Focal Nodular Hyperplasia, and Angio
Sarcoma. The bulk o f these are discussed in greater detail in the adult GI chapter.

369
Q

Choledochal cysts

A

are congenital dilations of the bile ducts -classified into 5 types by
some dude named Todani. The high yield trivia is type 1 is focal dilation o f the CBD and is by
far the most common. Type 2 and 3 are super rare. Type 2 is basically a diverticulum o f the
bile duct. Type 3 is a “choledochocele.” Type 4 is both intra and extra hepatic. Type 5 is
Caroli’s, and is intrahepatic only. I’ll hit this again in the GI chapter.

370
Q

Caroli’s

A

is an AR disease associated with polycystic kidney disease and medullary sponge
kidney. The hallmark is intrahepatic duct dilation, that is large and secular. Buzzword is
“central dot sign” which corresponds to the portal vein surrounded by dilated bile ducts.

371
Q

AR Polycystic Kidney Disease

A

This will be discussed in greater detail in the renal
section, but kids with AR polycystic kidney disease will have cysts in the kidneys, and
variable degrees of fibrosis in the liver. The degree o f fibrosis is actually the opposite of
cystic formation in the kidneys (bad kidneys ok liver, ok kidneys bad liver).

372
Q

Hereditary Hemorrhagic Telangiectasia (Osler-Weber-Rendu):

A

Autosomal dominant disorder characterized by multiple AVMs in the liver and lungs. It leads
to cirrhosis, and a massively dilated hepatic artery. The lung AVMs set you up for brain
abscess.

373
Q

Biliary Atresia

A

If you have prolonged newborn jaundice (> 2 weeks) you should think
about two things (1) neonatal hepatitis, and (2) Biliary Atresia. It’s critical to get this
diagnosis right because they need corrective surgery (Kasai Procedure) prior to 3 months.
Patients with biliary atresia really only have atresia of the ducts outside the liver (absence of
extrahepatic ducts), in fact they have proliferation o f the intrahepatic ducts. They will
develop cirrhosis without treatment and not do well

374
Q

Trivia to Know about Biliary Atresia

A

• Associations with Polysplenia, and Trisomy 18
• Gallbladder may be absent (normal gallbladder - supports neonatal hepatitis)
• Triangle Cord Sign - triangular echogenic structure by the portal vein - possibly remnant of
the CBD.
• Hepatobiliary Scintigraphy with 99m Tc-IDA is the test o f choice to distinguish (discussed
in the Nukes Chapter).
• Alagille Syndrome: This is a total zebra. All you need to know is hereditary cholestasis,
from paucity o f intrahepatic bile ducts, and peripheral pulmonary stenosis. The purpose o f a
liver biopsy in biliary atresia is to exclude this diagnosis.

375
Q

Gallstones

A

If you see a peds patient with gallstones think sickle cell.

376
Q

Sickle Cell

spleen

A

These kids bodies have spleen better days - as the spleen will typically
enlarge progressively and then eventually auto-infarct and shrink (during the first decade). If
the spleen remains enlarged it can run into problems - mainly acute splenic sequestration
crisis.

377
Q

Sickle Cell

splenic sequestration

A

This is the second most common cause o f death in SC patients
younger than 10. We are talking about the situation in which the spleen becomes a greedy
little pig and tries to hog all the blood for itself.

378
Q

Sickle Cell spleen

gamesmanship splenic sequestration

A

History o f abdominal pain or vital signs suggesting low volume (high HR,
low BP) with a big spleen. Remember most kids with sickle cell will have smaller spleens
(auto infarct) so a big spleen should be your clue.

379
Q

Sickle Cell

other problems if your spleen stays too big

A

Other problems you can run into if your spleen stays big are abscess formation and large
infarcts. These large infarcts are not the same pathophysiology as the “auto-infarct” you
typically think of with sickle cell. These are the big wedge shaped infarcts (hypo-perfusion on
CT). As a point of trivia, infarcted splenic tissue should look hypoechoic on US, with linear
“bright bands ” — google that if you haven’t seen it before.

380
Q

Sickle Cell

autoinfarcted spleen

A

This is different than the massive infarct in that it is typically the
combined effort of numerous tiny, unnoticeable, and repetitive micro occlusions leading to
progressive atrophy. Supposedly this doesn’t hurt (large infarcts do). This tends to occur
early and is usually “complete” by age 8. The typically look is going to be a tiny (possibly
calcified) spleen. When I say tiny - we are talking like 1cm. In the imaginary world of
multiple choice you might not even see the fucking thing

381
Q

cant see the spleen

A

Auto Infarct = Sickle Cell

382
Q

Sickle Cell

too small spleen gamesmanship

A

If you don’t see the spleen but you do see a gallbladder full o f stones in a kid
less than 15 - you should think Sickle Cell.

383
Q

Polysplenia and Asplenia

A

Heterotaxia syndromes are clutch for multiple choice

tests. The major game played on written tests is “left side vs right side. ”

384
Q

Heterotaxia syndromes

So what the hell does that mean

A

Let me break this down like a cardboard box that you intend
to put in your recycling … because this gets pretty fucking complicated. I like to start in the
lungs. The right side has two fissures (major and minor). The left side has just one fissure.
So if 1 show you a CXR with two fissures on each side, (a left sided minor fissure), then the
patient has two right sides. Thus the term “bilateral right sidedness.”

385
Q

Heterotaxia syndromes

What else is a right sided
structure?

A

The liver. So,
these patients won’t have a
spleen (the spleen is a left
sided structure).

386
Q

Heterotaxia syndromes

The opposite is true.

A

Since the spleen is on the left, a
“bilateral left sided” patient
will have polysplenia.

387
Q

Heterotaxia syndromes

Aorta/IVC

A

This relationship is a little more confusing when you try to reason it out. The
way I keep it straight is by remembering that the IVC is usually on the right. If you are
“bilateral left” then you don’t have a regular IVC — hence the azygos continuation. Then
just remember that the other one (flipped IVC/Aorta) is the other one.

388
Q

Right Isomerism

A

Reversed (Aorta/ ICV)

-Asplenia

389
Q

Left Isomerism

A

Azygos Continuation

-Polysplenia

390
Q

H e te ro ta x ia Syndromes

right sided

A

two fissures in left lung

asplenia

increased cardiac malformations

reversed aorta/ivc

391
Q

H e te ro ta x ia Syndromes

left sided

A

one fissure in right lung

polysplenia

less cardiac malformations

azygos continuation of the ivc

392
Q

Situs Vocab

A

• Situs Solitus - Instead o f just saying normal, you can be an asshole and say “Situs Solitus. ”
• Situs Inversus Totalis - Total mirror image transposition o f the abdominal and thoracic
stuff.
• Situs Ambiguus (Ambiguous) - This is a tricky way of saying Heterotaxy, o f which you can
have left or right “isomerism.”

393
Q

“Isomerism”

A

1 guess some asshole really liked organic chemistry…. This is a fancy
way of saying bilateral right or bilateral left

394
Q

Situs Solitus

Normal

A
• Gastric Bubble
on Left
• Larger part of
Liver on Right
• Minor fissure on
Right
395
Q

Situs

Inversus

A
• Gastric
Bubble on
Right
• Larger part of
Liver on Left
• Minor fissure
on Left
• Inverted
Bronchial
Pattern
• Associated
with Primary
Ciliary
Dyskinesia
396
Q

V.H. with T
Left
Isomerism

A
• Absent Minor
Fissures
• Interrupted
I VC
• Polysplenia
• Biliary
Atresia
( 10%)
397
Q

Top 4 Things for Peds Gl

biliary atresia

A

• Congenital liver fibrosis -» cholangiopathy
-» neonatal jaundice (after 1 week of life)
• US: Bright band of tissue (triangular cord
sign) near branching of common bile duct;
small or absent gallbladder (fasting ~ 3
hours)
• Scintigraphic: No tracer excretion into bowel
by 24 hours
• Biopsy to exclude = Zebra Alagille syndrome
• Treated with Kasai procedure

398
Q

Top 4 Things for Peds Gl

malrotation

A
• Duodenum to the right of the midline
• Increased risk for mid gut volvulus, and
internal hernia
• “Bilious vomiting”
• SMA to the right of the SMV
399
Q

Top 4 Things for Peds Gl

heterotaxia syndromes

A

Right Sided:
two fissures in left lung

asplenia

increased cardiac malformations

reversed aorta/ivc

LEft sided:
one fissure in right lung

polysplenia

less cardiac malformations

azygos continuation of the ivc

400
Q

Top 4 Things for Peds Gl

vacterl

A
V - Vertebral Anomalies (37%)
A - Anal (imperforate anus) (63%)
C - Cardiac (77%)
TE - Tracheoesophageal Fistula
or Esophageal Atresia ( 40%)
R - Renal (72%)
L - Limb (radial ray) - 58%
401
Q

Renal Agenesis

Most likely way to test this

A

Show unilateral agenesis on prenatal US - as an absent renal artery (in
view of the aorta) or oligohydramnios - with followup questions on associations. Or make it super
obvious with a CT / MRI and ask association questions.

402
Q

Renal Agenesis

Unilateral Absence Association

A

• 70% of women with unilateral renal agenesis have associated genital anomalies (usually
unicornuate uterus, or a rudimentary horn).
• 20% of men are missing the epididymis, and vas deferens on the same side they are missing the
kidney. PLUS they have a seminal vesicle cyst on that side.

403
Q

Renal Agenesis

Potter Sequence

A

Insult (maybe ACE inhibitors) = kidneys don’t form, if kidneys don’t form you can’t
make piss, if you can’t make piss you can’t develop lungs (pulmonary hypoplasia).

404
Q

Renal Agenesis

Lying Down Adrenal or “Pancake Adrenal” Sign

A

describes the elongated appearance of the adrenal
not normally molded by the adjacent kidney. It can be used to differentiate surgical absent vs
congenitally absent.

405
Q

Horseshoe Kidney

A

This is the most common fusion anomaly. The kidney gets hung up on the
IMA. Questions are most likely to revolve around the complications / risks:
• Complications from Position - Easy to get smashed against vertebral body - kid shouldn’t play
football or wrestle.
• Complications from Drainage Problems: Stones, Infection, and Increased risk of Cancer (from
chronic inflammation) - big ones are Wilms, TCC, and the Zebra Renal Carcinoid.
• Association Syndrome Trivia - Turner’s Syndrome is the classic testable association.

406
Q

Crossed Fused Renal Ectopia

A

One kidney comes across the midline and fuses with the
other. Each kidney has its own orthotopic ureteral
orifice to drain through. It’s critically important to the
patient to know that “the Ectopic Kidney is
Inferior. ” The left kidney more commonly crosses
over to the right fusing to the normal right kidney
lower pole. Complications include stones, infection,
and hyponephrosis (50%).
The classic way to show this is two axial CTs. The
first at the level of the kidneys hinting that one
kidney may be absent. The second through the
bladder (on a delayed phase) showing two opacified ureters.

407
Q

Congenital UPJ Obstruction

A

This is the most common congenital anomaly of the GU tract in neonates. About 20% of the time,
these are bilateral. Most (80%) of these are thought to be caused by intrinsic defects in the circular
muscle bundle of the renal pelvis. Treatment is a pycloplasty. A Radiologist can actually add value

408
Q

Congenital UPJ Obstruction

1970 called and they want to know how to tel! the difference between a prominent extrarenalpelvis
v.v a congenital UPJ obstruction

A

Whitaker Test ”, which is a urodynamics study combined with an antegrade pyelogram.

409
Q

Congenital UPJ Obstruction

Classic history

A

Teenager with flank pain after drinking “lots of fluids.”

410
Q

Congenital UPJ Obstruction

classic trivia

A

These do NOT have dilated ureters (NO HYDROURETER).

411
Q

Autosomal Recessive Polycystic Kidney

Disease (ARPKD)

A

These guys get HTN and renal
failure. The liver involvement is different than the adult
form (ADPKD). Instead of cysts they have abnormal
bile ducts and fibrosis. This congenital hepatic fibrosis is
ALWAYS present in ARPKD. The ratio of liver and
kidney disease is inverse. The worse the liver is the
better the kidneys do. The better the liver is the worse
the kidneys arc. Death is often from portal hypertension.

412
Q

Autosomal Recessive Polycystic Kidney
Disease (ARPKD)

ultrasound

A

On ultrasound the kidneys are smoothly enlarged and diffusely echogcnic, with a loss of corticomedullary
differentiation. In utero you sometimes will not see
urine in the bladder. Cysts tend to be tubular and spare
the cortex

413
Q

ARPKD us quick

A

Big Bright, with Lost

Corticomedullary Differentiation.

414
Q

Neonatal Renal Vein Thrombosis

A

This is an associated condition of maternal diabetes,
sepsis, and dehydration. It is typically unilateral (usually left). The theory is that it starts peripherally
and progresses toward the hilum. When acute, will cause renal enlargement. When chronic, will
result in renal atrophy.

415
Q

Neonatal Renal Artery Thrombosis

A

This occurs secondary to umbilical artery

catheters. Unlike renal vein thrombosis it does NOT present with renal enlargement but instead severe hypertension.

416
Q

Prune Belly (Eagle Barrett Syndrome)

A

This is a malformation triad which occurs in males. Classically
shown on a babygram with a kid shaped like a pear (big wide belly).

417
Q

Prune Belly (Eagle Barrett Syndrome)

triad

A

• Deficiency of abdominal musculature
• Hydroureteronephrosis
• Cryptorchidism
(bladder distention interferes with descent o f testes)

418
Q

Congenital (primary) MEGAureter

A

This is a “wastebasket” term for an enlarged
ureter which is intrinsic to the ureter (NOT the result of a distal obstruction). Causes include
(1) distal adynamic segment (analogous to achalasia, or colonic Hirschsprungs), (2) reflux at
the UVJ, (3) it just wants to be big (totally idiopathic). The distal adynamic type “obstructing
primary megaureter,” can have some hydro, but generally speaking an absence o f dilation o f
the collecting system helps distinguish this from an actual obstruction.

419
Q

Retrocaval Ureter (circumcaval)

A

This is actually a
problem with the development o f the IVC, which grows in a
manner that pins the ureter. Most o f the time it’s asymptomatic,
but can cause partial obstruction and recurrent UTI. I VP will
show a “reverse J” or “fishhook” appearance o f the ureter.

420
Q

Duplicated System

A

The main thing to know
about duplicated systems is the so-called “Weigert-
Meyer Rule” where the upper pole inserts inferior and
medially. The upper pole is prone to ureterocele
formation and obstruction. The lower pole is prone to
reflux. Kidneys with duplicated systems tend to be
larger than normal kidneys. In girls, a duplicated
system can lead to incontinence (ureter may insert
below the sphincter - sometimes into the vagina).

421
Q

Duplicated system

upper v lower

A
  • Upper Pole Obstructs
  • Lower Pole Refluxes
Upper Pole
(obstructed /
dilated) inserts
Inferior and
Medial

Lower Pole
inserts Superior
and Lateral

422
Q

Ureterocele

A

A cystic dilation o f the intravesicular
ureter, secondary to obstruction at the ureteral orifice. IVP
(or US) will show the “cobra head” sign, with contrast
surrounded by a lucent rim, protruding from the contrast
filled bladder. This is associated with a duplicated system
(specifically the upper pole).

423
Q

Ectopic Ureter

A

The ureter inserts distal to the external sphincter in the vestibule. More
common in females and associated with incontinence (not associated with incontinence in
men). Ureteroceles are best demonstrated during the early filling phase o f the VCUG.

424
Q

Posterior Urethral Valves

A

This is a fold in the posterior urethra that leads to
outflow obstruction and eventual renal failure (if it’s
not fixed). It is the most common cause of urethral
obstruction in male infants.

425
Q

Posterior Urethral Valves

trivia

A

The fold is a Wolfian Duct tissue remnant

426
Q

Posterior Urethral Valves

classic VCUG

A

The key finding on VCUG is an
abrupt caliber change between the dilated posterior
urethra and normal caliber anterior urethra.

427
Q

Posterior Urethral Valves

Fetal MRI

A

The MR1 would have to show hydro in

the kidney and a “key-hole” bladder appearance.

428
Q

Posterior Urethral Valves

Prenatal ultrasound

A
  • Hydronephrosis
  • Bladder Dilation
  • Oligiohydramnios
429
Q

Posterior Urethral Valves

buzzword

A

“Peri-renal fluid collection” is a buzzword, and it’s
the result o f fomiceal rupture. Obviously that is
non-specific and can be seen with any obstructive pathology.

430
Q

Non-Obstructive Causes of

Hydro in Baby Boys

A
  • Vesicoureteral Reflux (VUR)
  • Primary Megaureter
  • Prune Belly = Zebra
431
Q

Obstructive Causes of

Hydro in Baby Boys

A
  • PUV
  • UPJ Obstruction
  • Ureteral Ectopia
432
Q

Hydro workup

A

Hydronephrosis on Routine Prenatal Screening > repeat ultrasound once born > goes away in most > if persistent > VCUG to ealuate anatomy (gold standard), adjunctive MAG 3 to evaluate function and drainage (obstructive vs non obstructive), MR urography offers functionand structure but requires sedation in little kids

433
Q

Vesicoureteral Reflux (VUR)

A

Normally, the ureter enters the bladder at an
oblique angle so that a “valve” is developed.
If the angle of insertion is abnormal
(horizontal) reflux can develop. This can
occur in the asymptomatic child, but is seen
in 50% o f children with UTIs. The
recommendations for when the boy/girl with
a UTI should get a VCUG to evaluate for
VUR is in flux (not likely to be tested). Most
of the time VUR resolves by age 5-6.

434
Q

Vesicoureteral Reflux (VUR)

trivia

A

Hydronephrosis is the most common

cause of a palpable renal “mass” in childhood.

435
Q

Vesicoureteral Reflux (VUR)

grading system

A

There is a grading system fo r VUR which goes 1-5.

One is reflux halfway up the ureter,

Two is reflux into a non-dilated collecting system, (calyces still pointy),

Three you have dilation o f the collecting system, and calyces get blunted

Four the system gets mildly tortuous,

Five the system is very tortuous

436
Q

Vesicoureteral Reflux (VUR)

a sneaky trick

A

A sneaky trick would be to show the echogenic mound near the UVJ, that results from
injection o f “deflux”, which is a treatment urologist try. Essentially, they make a bubble with
this proprietary compound in the soft tissues near the UVJ and it creates a valve (sorta).
Anyway, they show it in a lot o f case books and textbooks so just like a midget using a urinal -
remember to stay on your toes.

437
Q

Vesicoureteral Reflux (VUR)

pearl

A

Chronic reflux can lead to scarring. This scarring can result in
hypertension and/or chronic renal failure

438
Q

Vesicoureteral Reflux (VUR)

additional pearl

A

If the reflux appears to be associated with a “hutch” diverticulum - people
will use the vocabulary “Secondary” VUR rather than Primary VUR. The treatment in this
case will be surgical. Ureteroceles, Posterior Valves, Neurogenic Bladder - are all causes of
Secondary VUR.

439
Q

Vesicoureteral Reflux (VUR)

normal

A

-Intravesicular

ureter is oblique

440
Q

Vesicoureteral Reflux (VUR)

reflux

A

intraversicular ureter is horizontal

441
Q

“Hutch” Diverticula

A
  • Occur at or adjacent (usually just above) the UVJ.
  • Caused by congenital muscular defect
  • Difficult to see on US — better seen with VCUG.
  • They are “dynamic” and best seen on the voiding (micturition)
  • If associated with VUR will often be surgically resected
442
Q

The Urachus

A

The umbilical attachment to the bladder (started out being called the allantois, then
called the Urachus). This usually atrophies into the umbilical ligament. Persistent canalization can
occur along a spectrum (patent, sinus, diverticulum, cyst).

443
Q

The Urachus

triiva 1

A

Most common complication of

urachal remnant = infection

444
Q

The Urachus

trivia 2

A

Urachal anomalies are twice as common in boys

relative to girls

445
Q

The Urachus

trivia 3

A

The most important piece of trivia is that when
these guys get cancer, it’s adenocarcinoma (90% of
cases). To hint at this multiple choice test writers will
often use the phrase “midline bladder structure

446
Q

urachal spectrum

A

Normal Obliterated Urachus (only the ligament remains)

Patent Urachus

Vesicourachal Diverticulum

Urachal Cyst

Umbilical Urachal Sinus

447
Q

Bladder Exstrophy

A

This is a herniation of the urinary

bladder through a hole in the anterior infra-umbilical abdominal wall.

448
Q

Bladder Exstrophy

what to know

A

• Increased incidence of Malignancy in the extruded bladder
• It’s Adenocarcinoma - just like a Urachal Remnant would
get
• Aunt Minnie “Manta Ray Sign” - with unfused pubic
bones. This looks like a monster wide pubic symphysis on
an AP pelvic radiograph.

449
Q

Cloacal Malformation

A

gu and gi both drain into a common openeing (like a birk) this only happens in females

450
Q

Neurogenic Bladder

A

1 will discuss this more in the adult Urinary chapter, but for kids I want you
to think about spinal dysraphism (tethered cord, sacral agenesis, and all the other fucked up spine stuff).

451
Q

Renal mass quick

neonate

A

nephroblastomasosis

mesoblastic nephroma

452
Q

Renal mass quick

around age 4

A

wilms

wilms variants

lymphoma

multilocular cystic nephroma

453
Q

Renal mass quick

teenager

A

rcc

lymphoma

454
Q

Rapid Review Trivia

mesoblastic nephroma

A

Solid Tumor o f Infancy ” (you can be born with it)

455
Q

Rapid Review Trivia

nephroblastomatosis

A

“Nephrogenic Rests” - left over embryologic crap that didn’t go away
Might turn into wilms (bilateral wilms especially)
“Next Step” - f/u ultrasound till 7-8 years old
Variable appearance

456
Q

Rapid Review Trivia

wilms

A

90% + Renal Tumors
“Solid Tumor o f Childhood” - Never born with it
Grows like a solid ball (will invade rather than incase)
Met to the lung (most common)

457
Q

Rapid Review Trivia

clear cells wilms

A

met to bone

458
Q

Rapid Review Trivia

rhabdoid wilms

A

Brain Tumors

It fucks you up, it takes the money (it believes in nothing Lebowski)

459
Q

Rapid Review Trivia

multi cystic nephroma

A

Micheal Jackson Tumor (Young Boys, Middle Age Women)
Big cysts that don’t communicate
Septal Enhancement
Can’t Tell it is not Cystic Wilms (next step = resection)

460
Q

Rapid Review Trivia

rcc

A

“Solid Tumor o f Adolescent”

Syndromes - VHL, TS

461
Q

rapid review trivia

renal lymphoma

A

Non-Hodgkin

Multifocal

462
Q

Solid Age renal masses 0-3

A

Nephroblastomatosis

Mesoblastic Nephroma

463
Q

Nephroblastomatosis

A

These are persistent nephrogenic rests beyond 36
weeks. It’s sorta normal (found in 1% of
infants). But, it can be a precursor to Wilms
so you follow it. When Wilms is bilateral,
99% of the time it had nephroblastomatosis
first. It goes away on its own (normally).
It should NOT have necrosis - this makes
you think Wilms. It has a variable
appearance, and is often described as
“homogeneous.” Although more commonly
a focal homogeneous ball, the way it’s
always shown in case conferences and case
books is as a hypodense rind.

Ultrasound screening q 3 months till age 7-8 is the usual routine - to make sure it doesn’t go
Wilms on you.

-Hypodense Rind -

464
Q

Mesoblastic Nephroma

A

“Solid renal tumor o f infancy.” This is a fetal hamartoma,
and generally benign. It is the most common neonatal renal tumor (80% diagnosed in the first
month on life). Often involves the renal sinus. Antenatal ultrasound may have shown
polyhydramnios.
Pearl: If it really looks like a Wilms, but they are just too young (< 1 year) then call it mesoblastic nephroma.

465
Q

Cystic Age renal mass 0-3

A

Multicystic Dysplastic Kidney

466
Q

Multicystic Dysplastic Kidney

A

You have multiple tiny cysts forming in utereo.
What you need to know is (1) that there is “no functioning renal tissue,” (2) contralateral renal
tract abnormalities occur like 50% o f the time (most commonly UPJ obstruction).

467
Q

Multicystic Dysplastic Kidney

mcdk vs bad hydro

A
  • In hydronephrosis, the cystic spaces are seen to communicate.
  • In difficult cases renal scintigraphy can be useful. MCDK will show no excretory function.
468
Q

Multicystic Dysplastic Kidney

pearl

A

MCDK has MACROscopic cysts that do NOT communicate

469
Q

Solid renal mass Around Age 4

A

Wilms

470
Q

Wilms

associated sydromes overgrowth

A

• Beckwith-Wiedemann - Macroglossia (most common
finding), Omphalocele, Hemihypertrophy, Cardiac, Big Organs.
• Sotos - Macrocephaly, Retarded (CNS stuff), Ugly Face

471
Q

Wilms

associated sydromes nonovergrowth

A

• WAGR - Wilms, Aniridia, Genital, Growth Retardation
• Drash - Wilms, Pseudohermaphroditism, Progressive
Glomerulonephritis

472
Q

Wilms

bilateral

A

About 5-10% will have bilateral disease ( “Synchronous Bilateral Wilms ”)

473
Q

Wilms variants

A

(look ju s t like Wilms)
• Clear Cell - likes to go to bones (lytic)
• Rhabdoid - “Terrible Prognosis” - Associated with aggressive Rhabdoid brain tumors

474
Q

Wilms nevers

A

• NEVER Biopsy suspected Wilms
(you can seed the tract and up the stage)
• Wilms NEVER occurs before 2 months of age
(Neuroblastoma can)

475
Q

Wilms in a 1 year old ?

A

Think about associated syndromes. Wilms loves to pal around with:
• Hemihypertrophy,
• Hypospadias,
• Cryptorchidism

476
Q

I Say Beckwith-

Wiedemann

A

You Say,
•Wilms,
•Omphalocele,
•Hepatoblastoma

477
Q

Cystic renal mass Around Age 4

A

Multilocular Cystic Nephroma

478
Q

Multilocular Cystic Nephroma

A

“Non-communicating, fluid-tilled locules,
surrounded by thick fibrous capsule.” By
definition these things are characterized by
the absence of a solid component or necrosis.

There is a classic bimodal occurrence
(4 year old boys, and 40 year old women).
I like to think o f this as the Michael Jackson
lesion - it loves young boys and middle-aged
women

479
Q

Multilocular Cystic Nephroma

buzzword

A

“protrude into the renal pelvis.”

480
Q

Multilocular Cystic Nephroma

turbonerds

A

One of my favorite jokes has been this “Michael Jackson lesion” to help remember
the age distribution. Unfortunately - a bunch of Academic Nerds got together and
decided that the pediatric cystic nephromas arc their own thing now, and the adult
ones are their own thing - with the words “adult cystic nephromas” and “pediatric
cystic nephroma” as the preferred nomenclature. Assholes…. ruined a great joke.

481
Q

Multilocular Cystic Nephroma

gamesmanship

A

Board exams usually lag a few years behind these kinds of changes. So if you get
a question asking about the age distribution, you may want to still go with the bimodal occurrence -
especially if they don’t say “adult” or “peds” — just make sure there isn’t another more correct
answer. This is where reading all the choices is critically important. Mind reading is also helpful.
Don’t forget to read the mind of the person who wrote the question - so you can understand his or
her bias.

482
Q

Solids renal mass in T e e n a ger

A

Renal Lymphoma and RCC can occur in teenagers. Renal lymphoma can occur in 5 year olds as well.

483
Q

Solids renal mass in T e e n a ger

A

Renal Lymphoma and RCC can occur in teenagers. Renal lymphoma can occur in 5 year olds as well.

484
Q

Rhabdomyosarcoma

A

This is the most common bladder cancer in humans less than 10
years of age. They are often infiltrative, and it’s hard to tell where they originate from.
“Paratesticular Mass” is often a buzzword. They can met to the lungs, bones, and nodes.
The Botryoid variant produces a polypoid mass, which looks like a bunch of grapes. I’ll discuss
this again in the testicle section.

485
Q

Neuroblastoma

A

Isn’t a Renal Mass, but is frequently contrasted with Wilms so I want to
discuss it in the renal section. It is the most common extra-cranial solid childhood malignancy.
They typically occur in very young kids (you can be born with this). 95% of cases occur
before age 10. They occur in the abdomen more than the thorax (adrenal 35%,
retroperitoneum 30%, posterior mediastinum 20%, neck 5%).

486
Q

Neuroblastoma

staging

A

Things that up the stage include
crossing the midline, and contralateral positive
nodes. These things make it Stage 3.

487
Q

Neuroblastoma

better prognosis

A

Diagnosis in Age < 1, Thoracic Primary, Stage 4S.

488
Q

Neuroblastoma

associations

A
  • NF-1, Hirschsprungs, DiGeorge, Beckwith Wiedemann

* Most are sporadic

489
Q

Neuroblastoma

random trivia

A

• Opsomyoclonus (dancing eyes, dancing feet) - paraneoplastic syndrome associated with
neuroblastoma.
• “Raccoon Eyes” is a common way for orbital neuroblastoma mets to present
• MIBG is superior to Conventional Bone Scan for Neuroblastoma Bone Mets
• Neuroblastoma bone mets are on the “lucent metaphyseal band DDx”
• Sclerotic Bone mets are UNCOMMON
• Urine Catecholamines are always (95%) elevated

490
Q

Neuroblastoma

stage 4s high yield

A

• Less than 1 year old
• Distal Mets are Confined to Skin, Liver, and
Bone Marrow
• Excellent Prognosis.
**A common distractor is to say 4S goes to
cortical bone. This is false! It’s the marrow.

491
Q

Neuroblastoma

quick

A

Age: usually less than 2 (can occur in utero)

calcifies 90%

encases vessels (doesnt invade)

poorly marginated

mets to bones

492
Q

Wilms

quick

A

Age: Usually around age 4 (never before 2 months)

Calcifies Rarely (<10%)

Invades Vessels (doesn’t encase)

Well Circumscribed

Doesn’t usually met to bones (unless clear cell
Wilms variant). Prefers lung.

493
Q

Neonatal Adrenal Hemorrhage

A

This can occur in the setting o f birth

trauma or stress.

494
Q

Neonatal Adrenal Hemorrhage

trivia

A

Neonatal adrenal hemorrhage is associated with scrotal hemorrhage

495
Q

THIS v.s THAT: Hemorrhage VS Neuroblastoma:

A

• Ultrasound can usually tell the difference (adrenal hemorrhage is anechoic and
avascular, neuroblastoma is echogenic and hyper-vascular).
• MRI could also be done to problem solve if necessary (Adrenal Hemorrhage low T2 ,
Neuroblastoma high T2).

496
Q

Gamesmanship - “Next Step” Adrenal mass of a neonate

A

Neonates that are sick enough to be in the hospital hemorrhage their fucking adrenals all the
time. An adrenal hemorrhage can look just like a mass on ultrasound. Yes, technically it
should be anechoic and avascular - but maybe your tech sucks, or maybe you don’t get shown
a picture they just tell you it’s a mass. The question writer is most likely going to try and
trick you into worrying about a neuroblastoma (which is also going to be a mass in the
adrenal).

497
Q

Gamesmanship - “Next Step” Adrenal mass of a neonate

next step

A

Sticking a needle in it, sedating the kid for MRI, or exposing him/her to the
radiation o f CT, PET, or MIBG are all going to go against the “image gently” propaganda
being pushed at academic institutions. Plus it’s unnecessary. As is true with most things in
radiology, they either get better or they don’t. Hemorrhage is going to resolve. The cancer is
not. So the first step is going to be followup ultrasound imaging

498
Q

Gamesmanship - History of “adrenal insufficiency” ?

Does that help you in the setting o f a newborn adrenal mass?

A

Nope. Most cases are actually caused by a 21 alpha hydroxylase deficiency (congenital
adrenal hyperplasia). Those tend to look different than hemorrhage or a mass- they are more
“cerebriform.” The problem is you can acquire adrenal insufficiency from neoplastic
destruction (neuroblastoma) or regular good old fashioned hemorrhage.

499
Q

Hydrometrocolpos

A

Essentially the vagina won’t drain the uterus. This
condition is characterized on imaging by an expanded
fluid-filled vaginal cavity with associated distention of
the uterus.

500
Q

Hydrometrocolpos

presentation

A

You can see it presenting in infancy as a mass, or as a

teenager with delayed menarche.

501
Q

Hydrometrocolpos

causes

A

Causes include imperforate hymen (most common),
vaginal stenosis, lower vaginal atresia, and cervical
stenosis

502
Q

Hydrometrocolpos

testing

A

For multiple choice trivia think about this as a “midline
pelvic mass” , which can cause hydronephrosis (mass
effect from distended uterus).

503
Q

Hydrometrocolpos

trivia

A

Trivia: Associated with Uterus Didelphys (which often

~75% has a transverse vaginal septum)

504
Q

ovarian torsio

A

In an adult, ovarian torsion is almost always due to a mass. In a child, torsion can
occur with a normal ovary, secondary to excessive mobility o f the ovary. As described in the
GYN chapter you are going to see an enlarged (swollen) ovary, with peripheral follicles, with or
without arterial flow. What is “enlarged” ? Unlike an adult you can’t really use a fixed number
to call the ovary enlarged (ovarian volumes in the peds setting are notoriously variable). The
solution is to compare the ovary in question to the contralateral size. Suspect torsion if the ovary
is at least 3 times the size of the opposite “normal” ovary. Fluid-Debris Levels within the
displaced follicles is another described adolescent ovarian torsion finding.

505
Q

ovarian masses

A

About two-thirds of ovarian neoplasms are benign dermoids/teratomas (discussed in
detail in the GYN chapter). The other one third are cancer. The cancers are usually germ cells
(75%). Again, mural nodules and thick septations should clue you in that these might be cancer.
Peritoneal implants, ascites, and lymphadenopathy, are all bad signs and would over-ride
characteristics of the mass.

506
Q

Hydrocele

A

Collection of serous fluid and is the most common cause of painless scrotal
swelling. Congenital hydroceles result from a patent processus vaginalis that permits entry of
peritoneal fluid into the scrotal sac.

507
Q

Complicated Hydrocele (one with septations):

A

This is either a hematocele vs pyocele. The distinction is clinical.

508
Q

Varicocele

A

Most of these are idiopathic and found in adolescents and young adults. They are
more frequent on the left. They are uncommon on the right, and if isolated (not bilateral) should
stir suspicion for abdominal pathology (nutcracker syndrome, RCC, retroperitoneal fibrosis).

509
Q

Varicocele next step

A

Isolated right-sided varicocele = Abdomen CT
Exclude the extrinsic mass, renal vein thrombus, or
portal hypertension causing a splenorenal shunt

510
Q

Scrotal HSP

A

This vasculitis is the most common cause of idiopathic scrotal edema

511
Q

Acute Pain in or around the Scrotum

A

The top three considerations in a child with acute scrotal pain are (1) torsion of the testicular
appendage, (2) testicular torsion, and (3) epididymo-orchitis.

512
Q

Epididymitis

A

The epididymal head is the most common part involved. Increased size and
hyperemia are your ultrasound findings. This occurs in two peaks: under 2 and over 6. You can
have infection of the epididymis alone or infection of the epididymis and testicle (isolated orchitis
is rare).

513
Q

Orchitis

A

Nearly always occurs as a progressed epididymitis. When isolated the answer is mumps.

514
Q

Torsion of the Testicular Appendages

A

This is the most common cause of acute
scrotal pain in age 7-14. The testicular appendage is some vestigial remnant of a mesonephric
duct. Typical history is a sudden onset of pain, with a Blue Dot Sign on physical exam (looks like
a blue dot). Enlargement of the testicular appendage to greater than 5 mm is considered by some
as the best indicator of torsion

515
Q

Torsion of the Testicle

A

Results from the testis and spermatic cord twisting within the
serosal space leading to ischemia. The testable trivia is that it is caused by a failure o f the tunica
vaginalis and testis to connect or a “Bell Clapper Deformity”. This deformity is usually bilateral,
so if you twist one they will often orchiopexy the other one. If it was 1950 you’d call in your
nuclear medicine tech for scintigraphy. Now you just get a Doppler ultrasound. Findings will be
absent or asymmetrically decreased flow, asymmetric enlargement, and slightly decreased
echogenicity of the involved ball.

516
Q

Paratesticular Rhabdomyosarcoma

A

By far the most common extra-testicular
mass in young men and the only one really worth mentioning. If you see a mass in the
scrotum that is not for sure in the testicle this is it (unless the history is kick to the balls from a
spiteful young lady - and you are dealing with a big fucking hematoma). If it’s truly a mass -
this is the answer.

517
Q

Paratesticular Rhabdomyosarcoma

trivia

A

• The most common location is actually the head/neck - specifically the orbit and
nasopharynx.
• There is a bimodal peak (2-4, then 15-17).

518
Q

Testicular Masses

A

Testicular Masses can be thought of as intratesticular or extratesticular. With regard to
intratesticular masses, ultrasound can show you that there is indeed a mass but there are no
imaging features that really help you tell which one is which. If the mass is extratesticular , the
most likely diagnosis is an embryonal rhabdomyosarcoma from the spermatic cord or epididymis

519
Q

Testicular Mircolithiasis

A

This appears as
multiple small echogenic foci within the testes.
Testicular microlithiasis is usually an incidental
finding in scrotal US examinations performed for
unrelated reasons. It might have a relationship with
Germ Cell Tumors (controversial). Follow-up in 6
months, then yearly is probably the recommendation
(maybe - it’s very controversial, and therefore unlikely to be asked).

520
Q

Testicular Cancer

A

Germ Cell (90%)
• Seminoma (40%) - seen more in the 4th decade
• Non Seminoma (60%)
• Teratoma, Yolk Sacs, Mixed Germ Cells, Etc.
Non Germ Cell (10%)
•Sertoli
•Leydig

521
Q

THIS vs THAT:
Testicular
Calcifications

A

Tiny (micro) = Seminoma

BIG = Germ Cell Tumor

522
Q

The two Germ Cell Tumors seen in the first decade of life are the

A

yolk sac tumor, and the

teratoma.

523
Q

Yolk Sac Tumor

A

Heterogeneous Testicular Mass in < 2 year old = Yolk Sac Tumor. AFP is
usually super elevated.

524
Q

Teratoma

A

Pure testicular teratomas are only seen in young kids < 2. Mixed teratomas are
seen in 25 year olds. Unlike ovarian teratoma, these guys often have aggressive biological behavior.

525
Q

Choriocarcinoma:

A

An aggressive, highly vascular tumor, seen more in the 2nd decade.

526
Q

Sertoli Cell Tumors

A

These testicular tumors are usually bilateral and are visualized on US
as “burned-out” tumors (dense echogenic foci that represent calcified scars). A subtype of Sertoli
cell tumor associated with Peutz-Jeghers syndrome typically occurs in children. If they show you
the Peurtz-Jegher lips and bilateral scrotal masses, this is the answer.

527
Q

Testicular Lymphoma

A

Just be aware that lymphoma can “hide” in the testes because of
the blood testes barrier. Immunosuppressed patients are at increased risk for developing
extranodal/ testicular lymphoma. On US, the normal homogeneous echogenic testicular tissue is
replaced focally or diffusely with hypocchoic vascular lymphomatous tissue. Buzzword =
multiple hypoechoic masses of the testicle.

528
Q

Sacrococcygeal Teratoma

A

This is the most common
tumor of the fetus or infant. These solid and/or cystic masses are
typically large and found either on prenatal imaging or birth.
Their largeness is a problem and can cause mass effect on the GI
system, hip dislocation, and even nerve compression leading to
incontinence. They are usually benign (80%), although those
presenting in older infants tend to have a higher malignant
potential. The location of the mass is either external to the pelvis
(47%), internal to the pelvis (9%), or dumbelled both inside and
outside (34%).

529
Q

Sacrococcygeal Teratoma

types

A

The easiest way to remember it is like this:

  • Type 1 - Totally extra pelvic
  • Type 2 - Barely pelvic, but not abdominal
  • Type 3 - Some abdominal
  • Type 4 - Totally inside abdomen ** this one has the highest rate o f malignancy.
530
Q

Sacrococcygeal Teratoma

trivia

A

They have to cut the coccyx off during resection. Incomplete resection of the coccyx is
associated with a high recurrence rate.

531
Q

Fracture

A

In general, little kids bend they don’t break.
You end up with lots of buckles and greensticks. For
problem solving you can get a repeat in 7-10 days as
periosteal reaction is expected in 7-10 days. Kids tend to
heal completely, often with no sign of prior fracture.

532
Q

Fracture

Involvement o f the Physis:

A

The major concern is growth
arrest, probably best asked by showing a physeal bar
(“early” bony bridge crossing the growth plate). You can
get bars from prior infection, but a history of trauma is
gonna be the more classic way to ask it.

533
Q

Salter-Harris Classification

Type 1

A

S - Slipped
Complete physeal fracture, with or without
displacement.

534
Q

Salter-Harris Classification

Type 2

A

A - Above (or “Away from the Joint”)
Fracture involves the metaphysis. This is the
most common type (75%).

535
Q

Salter-Harris Classification

Type 3

A

L - Lower
(3 is the backwards “E ”fo r Epiphysis)
Fracture involves the epiphysis. These guys have
a chance of growth arrest, and will often require
surgery to maintain alignment

536
Q

Salter-Harris Classification

Type 4

A

T - Through
Fracture involves the metaphysis and epiphysis.
These guys don’t do as well, often end up with
growth arrest, or focal fusion. They require
anatomic reduction and often surgery.

537
Q

Salter Harris Classification

Type 5

A

R - Ruined
Compression of the growth plate. It occurs from axial loading injuries, and has a very poor
prognosis. These are easy to miss, and often found when looking back at comparisons (hopefully
ones your partner read). The buzzword is “bony bridge across physis”.

538
Q

Toddler’s Fracture

A

Oblique fracture o f the midshaft o f the tibia seen in a child just
starting to walk (new stress on bone). If it’s a spiral type you probably should query nonaccidental
trauma. The typical age is 9 months - 3 years.

539
Q

Stress Fracture in Children

A

This is an injury which occurs after repetitive trauma,
usually after new activity (walking). The most common site o f fracture is the tibia - proximal
posterior cortex. The tibial fracture is the so-called “toddler fracture” described above. Other
classic stress fractures include the calcaneal fracture - seen after the child has had a cast
removed and returns to normal activity.

540
Q

The Elbow

A

Every first year resident knows that elevation o f the fat pad (sail sign) should make you think
joint effusion and possible occult fracture. Don’t forget that sometimes you can see a thin
anterior pad, but you should never see the posterior pad (posterior is positive). I like to bias
myself with statistics when I’m hunting for the peds elbow fracture. The most common
fracture is going to be a supracondylar fracture (>60%), followed by lateral condyle (20%),
and medial epicondyle (10%).

541
Q

RadiocapiteUar Line

A
This is a line
through the center of the radius, which
should intersect the middle of the
capitellum on every view (regardless of
position). If the radius is dislocated it
will NOT pass through the center of the
capitellum
542
Q

Anterior Humeral Line

A
This time you
need a true lateral. A line along the
anterior surface of humerus, should pass
through the middle third of the
capitellum. With a supracondylar
fracture (the most common peds elbow
fracture) you’ll see this line pass
through the anterior third.
543
Q

Ossification Centers are a
source of trickery.

elbow

A
Rr . emember they occur in a set order 
(c r it o e )
• Capitellum (Age 1)
• Radius (Age 3), 
• Internal (medial epicondyle Age 5),
• Trochlea (Age 7),
• olecranon(Age 9), 
• External (lateral epicondyle Age 11).
544
Q

Elbow Tricks

Lateral Condyle Fx

A

This is the second most common distal humerus fracture in kids.
Some dude named Milch classified them. The thing to know is a fracture that passes through
the capitello-trochlear groove is unstable (Milch II). Since it’s really hard to tell this,
treatment is based on the displacement of the fracture fragment (> or < 2mm).

545
Q

Elbow Tricks

Trochlea

A

can have multiple ossification centers, so it can have a fragmented appearance

546
Q

Elbow Tricks

Medial Epicondyle Avulsion (Little League Elbow

A

h e re are two major tricks
with this one. (1) Because it’s an extra-articular structure, its avulsions will not necessarily
result in a joint effusion. (2) It can get interposed between the articular surface of the
humerus and olecranon. Avulsed fragments can get stuck in the joint, even when there is no
dislocation.

547
Q

Elbow Tricks

Nursemaids Elbow:

A

When a child’s arm is pulled on, the radial head may sublux into
the annular ligament. X-rays typically don’t help, unless you supinate the arm during lateral
position (which often relocates the arm).

548
Q

Medial Epicondyle Avulsion (Little League Elbow

Anytime you see a dislocation - ask yourself

A
  • Is the patient 5 years old ? And if so

* Where is the medial epicondyle ?

549
Q

Medial Epicondyle Avulsion (Little League Elbow

The importance o f IT (crIToe)

A

• You should never see the trochlea and not see the
internal (medial epicondyle), if you do it’s probably a
displaced fragment

550
Q

common elbow fracturess

A

lateral condylar

medial epicondyle

551
Q

uncommon elbow fractures

A

lateral epicondyle

medial epicondyle

552
Q

Avulsion Injuries

iliac cres

A

abdominal muscles

553
Q

Avulsion Injuries

asis

A

sartorius

554
Q

Avulsion Injuries

aiis

A

rectus femoris

555
Q

Avulsion Injuries

greater trochanter

A

gluteal muscles

556
Q

Avulsion Injuries

lesser trochanter

A

iliopsoas

557
Q

Avulsion Injuries

ischial tuberosity

A

hamstrings

558
Q

Avulsion Injuries

symphysis

A

adductor

559
Q

Chronic Fatigue Injuries

A

Sinding-Larsen-Johansson

Osgood-Schlatter

560
Q

Sinding-Larsen-Johansson

A

This is a
chronic traction injury at the insertion of the patellar
tendon on the patella. It’s seen in active adolescents
between age 10-14. Kids with cerebral palsy are
prone to it.

561
Q

Osgood-Schlatter

A

This is due to repeated
micro trauma to the patellar tendon on its insertion at
the tibial tuberosity. It’s bilateral 25% of the time, and
more common in boys

562
Q

Periosteal Reaction in the Newborn

A
Congenital Rubella
Syphilis
Caffey Disease
Prostaglandin Therapy
Neuroblastoma Mets
Physiologic Growth
Abuse
563
Q

Congenital Rubella

A

Bony changes are seen in 50% of cases, with the classic buzzword
being “celery stalk” appearance, from generalized lucency of the metaphysis. This is usually
seen in the first few weeks o f life

564
Q

Syphilis

A

Bony changes are seen in 95% of cases.
Bony changes do NOT occur until 6-8 weeks o f life
(Rubella changes are earlier). Metaphyseal lucent
bands and periosteal reaction along long bones can be
seen. The classic buzzword is “Wimberger Sign” or
destruction of the medial portion of the proximal
metaphysis of the tibia.

565
Q

Caffey Disease

A

Have you ever seen that giant multiple volume set o f peds radiology
books? Yeah, same guy. This thing is a self limiting disorder of soft tissue swelling, periosteal
reaction, and irritability seen within the first 6 months of life. The classic picture is the
really hot mandible on bone scan. The mandible is the most common location (clavicle, and
ulna are the other classic sites). It’s rare as hell, and probably not even real. There have been
more sightings o f Chupacabra in the last 50 years.

566
Q

Prostaglandin Therapy

A

Prostaglandin El and E2 (often used to keep a PDA open) can
cause a periosteal reaction. The classic trick is to show a chest x-ray with sternotomy wires (or
other hints of congenital heart), and then periosteal reaction in the arm bones.

567
Q

Neuroblastoma Mets

A

This is really the only childhood malignancy that occurs in

newborns and mets to bones.

568
Q

Physiologic Growth

A

So this is often called “Physiologic Periostitis of the Newborn” ,
which is totally false and wrong. It does NOT happen in newborns. You see this around 3
months of age, and it should resolve by six months. Proximal involvement (femur) comes
before distal involvement (tibia). It always involves the diaphysis.

569
Q

Abuse

A

Some people abuse drugs, some just can’t stand screaming kids, some suffer both
shortcomings. More on this later.

570
Q

It is NOT physiologic periostitis if:

A
  • You see it before I month
  • You see it in the tibia before the femur
  • It does not involve the diaphysis.
571
Q

Langerhans Cell Histiocytosis (LCH)

A

Also known as EG (eosinophilic

granuloma). It’s twice as common in boys. Skeletal manifestations are highly variable

572
Q

Langerhans Cell Histiocytosis (LCH)

skull

A

Most common site. Has “beveled edge” from uneven destruction of the inner
and outer tables. If you see a round lucent lesion in the skull o f a child think this (and
neuroblastoma mets).

573
Q

Langerhans Cell Histiocytosis (LCH)

ribs

A

Multiple lucent lesions, with an expanded appearance

574
Q

Langerhans Cell Histiocytosis (LCH)

spine

A

Vertebra plana

575
Q

Osteomyelitis

A

It usually occurs in babies (30% o f cases less than 2 years old). It’s usually hematogenous
(adults it directly spreads - typically from a diabetic ulcer).
There are some changes that occur over time, which are potentially testable.

576
Q

Osteomyelitis

newborns

A

They have open growth plates and perforating vessels which travel from the
metaphysis to the epiphysis. Infection typically starts in the metaphysis (it has the most blood
supply because it is growing the fastest), and then can spread via these perforators to the
epiphysis.

577
Q

Osteomyelitis

kids

A

Later in childhood, the perforators regress and the avascular epiphyseal plate stops
infection from crossing over. This creates a “septic tank” scenario, where infection tends to
smolder. In fact, 75% of cases involve the metaphyses o f long bones (femur most common).

578
Q

Osteomyelitis

adults

A

When the growth plates fuse, the barrier of an avascular plate is no longer present,
and infection can again cross over to the epiphysis to cause mayhem.

579
Q

Osteomyelitis

trivia

A
  • Hematogenous spread more common in kids (direct spread in adult)
  • Metaphysis most common location, with target changes as explained above
  • Bony changes don’t occur on x-ray for around 10 days.
  • It’s serious business and can rapidly destroy the cartilage if it spreads into the joint
580
Q

Short Fingers

A

Brachydactyly

581
Q

Too Many Fingers

A

Polydactyly

582
Q

Two or More Fused Fingers

“Sock Hand” - 1 call it

A

Syndactyly

583
Q

Contractures o f Fingers

A

Camptodactyly

584
Q

Radially Angulated Fingers (Usually 5,h)

A

Clinodactyly

585
Q

Long, Spider-Like Fingers

A

Arachnodactyly

586
Q

Limb is Absent

A

Amelia

587
Q

Limb is mostly Absent

A

Meromelia

588
Q

Hands / Feet (distal limbs) are Short

A

Acromelic

589
Q

Forearm or Lower Leg are short (middle limbs)

A

Mesomelic

590
Q

Femur or Humerus (proximal limbs) are short

A

Rhizomelic

591
Q

Short All Over

A

Micromelic

592
Q

Achondroplasia

A

This is the most common skeletal dysplasia, and is the mostly likely
to be seen at the mall (or on television). It results from a fibroblast growth factor receptor
problem (most dwarfisms do). It is a rhizomelic (short femur, short humerus) dwarf. They
often have weird big heads, trident hands (3rd and 4th fingers are long), narrowing o f the
interpedicular distance, and the tombstone pelvis. Advanced paternal age is a risk factor.
They make good actors, excellent rodeo clowns, and various parts o f their bodies (if cooked
properly) have magical powers.

593
Q

Thanatophoric

A

This is the most common lethal dwarfism. They have rhizomelic
shortening (humerus, femur). The femurs are sometimes called telephone receivers. They
have short ribs and a long thorax, and small iliac bones. The vertebral bones are flat
(platyspondyly), and the skull can be cloverleaf shaped

594
Q

Asphyxiating Thoracic Dystrophy (Jeune)

A

This is usually fatal as well. The
big finding is the “Bell shaped thorax” with short ribs. 15% will have too many fingers
(polydactyly). If they live, they have kidney problems (chronic nephritis). You can
differentiate a dead Thanatophoric dwarf, from a dead Jeune dwarf by looking at their
vertebral bodies. The Jeune bodies are normal (the thanatophorics are flat).

595
Q

•Ellis-Van Crevald

A

is the dwarf with multiple fingers.

596
Q

•Pseudoachondroplasia

A

is this weird thing not present at birth, and spares the skull.

597
Q

Pyknodysostosis

A

osteopetrosis, in a dwarf with a wide angled jaw, & Acro-osteolysis

598
Q

The Dwarf Blitz - 5 Things I Would Remember About Dwarfs

A
  1. The Vocab: Rhizo (humerus, femur) vs Aero (hands, feet) vs Meso (forearm, tib/fib)
  2. Most dwarfs are Rhizomelic - if forced to choose, always guess this
  3. The pedicles are supposed to widen slightly as you descend the spinal column,
    Achondroplasia has the opposite - they narrow. If you see a live dwarf, with short
    femurs / humerus, and narrowing of the pedicles then this is the answer.
    (technically thantophorics can get this too - but it s more classic fo r achondroplasia)
  4. Thanatophoric is your main dead dwarf. Usually the standout feature is the
    telephone receiver femur (and a crazy cloverleaf head)
  5. Jeune is another dead dwarf - but the short ribs really stand out.
599
Q

Bifid Rib

A

This is the most common cause of an anterior wall “mass.” If there is just one (usually
the 4th rib) - then it is just a variant. If there are bunches think Gorlin Syndrome.

600
Q

Gorlin Syndrome

A

Bifid Ribs, Calcifications of the Falx, basal cell cancers, odontogeni keratocysts (lytic jaw lesions).

601
Q

Osteogenesis Imperfecta

A

They have a collagen defect and make brittle bones. Depending
on the severity it can be totally lethal or more mild. It’s classically shown with a totally lucent skull,
01 multiple fractures with hyperplastic callus. Another classic trick is to show the legs with the
fibula longer than the tibia. They have wormian bones, and often flat or beaked vertebral bodies.
Other trivia is the blue sclera, hearing impairment (otosclerosis), and that they tend to suck at football.

602
Q

Osteopetrosis

A

They have a defect in the way osteoclasts work, so you end up with
disorganized bone that is sclerotic and weak (prone to fracture). There are a bunch of different types,
with variable severity. The infantile type is lethal because it takes out your bone marrow. With less
severe forms, you can have abnormal diminished osteoclastic activity that varies during skeletal
growth, and results in alternating bands of sclerosis parallel to the growth plate. Most likely the way
this will be shown is the “bone-in-bone” appearance in the vertebral body or carpals. Picture frame
vertebrae is another buzzword. Alternatively, they can show you a diffusely sclerotic skeleton, with
diffuse loss of the corticomedullary junction in the long tubular bones.

603
Q

Pyknodysostosis

A

Osteopetrosis + Wormian Bones + Acro-Osteolysis. They also

have “wide (or obtuse) angled mandible ”, which apparently is a buzzword.

604
Q

Klippel Feil

A

You get congenital fusion of the cervical spine (sorta like JRA).
The cervical vertebral bodies will be tall and skinny. There is often a sprengel deformity
(high riding scapula). Another common piece of trivia is to show the omovertebral bone
- which is just some big stupid looking vertebral body.

605
Q

Hunters I Hurlers / Morquio

A

All three of these are mucopolysaccharidoses. Findings
include oval shaped vertebral bodies with anterior beak. The beak is actually mid in Morquio, and
inferior in Hurlers. Clavicles and ribs are often thick (narrow more medially) - like a canoepaddle.
The pelvis shape is described as the opposite of achondroplasia - the iliac wings are tall and
Haired. The hand x-ray is the most commonly shown in case books and gives you wide metacarpal
bones with proximal tapering.

606
Q

Few More Trivia Points on Morquio

A
  • They are dwarfs
  • The most common cause of death is cervical myelopathy at C2
  • The bony changes actually progress during the first few years of life
607
Q

Neurofibromatosis

A

Just briefly remember that type 1
can cause anterior tibial bowing, and pseudarthrosis at the
distal tibula.
This is an Aunt Minnie.
They often have scoliosis. Just think of the elephant man

608
Q

Gauchers

A

This is the most common lysosomal storage
disease. It gives you a big spleen, and big liver among a few
bone signs.
*A VN o f the Femoral Fleads
• FI-Shaped Vertebra
• Bone Infarcts (lots o f them)
•Erlenmeyer Flask Shaped Femurs

609
Q

Caudal Regression Syndrome

A

This is a spectrum
that includes sacral and/or coccyx agenesis. You see it with
VACTERL and Currarino Triads Syndromes.

610
Q

Scoliosis

A

Lateral curvature of the spine, which is usually idiopathic in girls. It can also be from
vertebral segmentation problems. NF can cause it as well (that’s a piece of trivia).

611
Q

Radial Dysplasia

A
Absence or hypoplasia of the radius (usually with a missing thumb) is a
differential case (VACTERL, Holt-Oram, Fanconi Anemia, Throbocytopenia Absent Radius). As a
point of trivia TAR kids will have a thumb.
612
Q

Hand Foot Syndrome

A

The classic history is hand or
foot pain / swelling in an infant with sickle cell. This is a
dactylitis, and felt to be related to ischemia. It will resolve on
its own, after a few weeks. Radiographs can show a
periostitis two weeks after the pain goes away.

613
Q

Blounts

A

tibia vara). Varus angulation occurring at the
medial aspect of the proximal tibia (varus bowing occurs at
the metaphysis not the knee). This is often bilateral, and
NOT often seen before age 2 (two sides, not before two).
Later in the disease progression the medial metaphysis will
be depressed and an osseous outgrowth classically develops.
You can see it in two different age groups; (a) early - which
is around age 2 and (b) late - which is around age 12.

Two Sides - Not Before Two
Two Different Ages (2-3, 12)

614
Q

Talipes

A

Congenita

615
Q

Pes

A

Foot or Acquired

616
Q

Equines

A

Plantar Flexed Ankle ”, Heel Cord is often tight, and the heel won’t touch the floor

617
Q

Calcaneus

A

Opposite o f Equines. The Calcaneus is actually angled up

618
Q

Varus

A

Forefoot in

619
Q

Valgus

A

Forefoot out

620
Q

Cavus

A

High Arch

621
Q

Planus

A

Opposite o f Cavus - “bizarro cavus” - FLAT FOOT

622
Q

Supination

A

Inward rotation - “Sole offoot in ” - holding soup with the bottom o f your foot

623
Q

Pronation

A

Outward rotation - “Sole offoot out”

624
Q

Hindfoot Valgus

A
Think about this as the talus
sliding nose down off the
calcaneus. This make the
angle wider.
If the talus slides off you lose
your longitudinal arch - which
essentially characterizes
hindfoot valgus.
Also, note that the nose
down (nearly vertical)
appearance of the talus .
“Too Many Toes”
625
Q

Talocalcaneal angle

A

hindfoot varus: <25°
normal: 25-40°
hindfoot valgus: >40°

626
Q

Hindfoot Varus

A
This is the opposite
situation, in which you
have a narrowing of the
angle between the talus
and calcaneus.
Notice the two bones lay
nearly parallel - like two
“clubs” laying on top of
each other.
627
Q

Flat Foot (Pes Planus)

A

This can be congenital or acquired. The peds section will cover
congenital and the adult MSK section will cover acquired. The congenital types can be grouped into
flexible or rigid (the flexible types are more common in kids). The distinction can be made with
plantar flexion views (flexible improves with stress). The ridged subtypes can be further subdivided
into tarsal coalition and vertical talus. In any case you have a hindfoot valgus

628
Q

Tarsal Coalition

A

There are two main types (talus to the calcaneus, and calcaneus to the
navicular). They are pretty equal in incidence, and about 50% o f the time are bilateral. You
can have bony or fibrous/cartilaginous subtypes. The fibrous/cartilaginous types are more
common than the bony types.

629
Q

Tarsal Coalition

Talocalcanea

A

Occurs at the middle facet.
Has the “continuous C-sign ” produced from an “absent middle fac et” on
the lateral view. Talar beak (spur on the anterior talus - white arrow) is also
seen in about 25% of cases.

630
Q

Tarsal Coalition

Calcaneonavicula

A

Occurs at the anterior facet.
Has the “anteater sign ” Where the elongated anterior process o f the
calcaneus resembles the blood thirsty nose of a ravenous ant eater. This
is best seen on an oblique view.

631
Q

Vertical Talus

A

(equinus hindfoot valgus) - This is
sometimes called the “rocker-bottom foot” because the talus is
in extreme plantar flexion with dorsal dislocation of the
Navicular - resulting in a locked talus in plantar flexion. As a
point of trivia this is often associated with myelomeningocele

632
Q

Club Foot (Talipes Equino Varus)-

A

Translation - Congenital Plantar Flexed Ankle
Forefoot. This is sorta why I lead with the vocab, all the congenital feet can be figured out based on
the translated language. This thing is more common in boys, and bilateral about half the time. The
toes are pointed down (equines), and the talocalcaneal angle is acute (varus).

633
Q

Club Foot (Talipes Equino Varus)-

key findings

A
* Hindfoot varus (decreased talocalcaneal
angle)
* Medial deviation and inversion of the
forefoot
* Elevated Plantar Arch
634
Q

Club Foot (Talipes Equino Varus)-

trivia

A

The most common surgical complication is over correction resulting in a “rocker bottom” flat foot deformity.

635
Q

Developmental Dysplasia of the Hip

overview

A

This is seen more commonly in females,
children bom breech, and oligohydramnios. The physical exam buzzwords are asymmetric
skin or gluteal folds, leg length discrepancy, palpable clunk, or delayed ambulation. It’s
bilateral about 1/3 of the time. Ultrasound is done to evaluate (after physical exam), and is
excellent until the bones ossify (then you need x-rays). A common trick is to be careful
making a measurement in the first week o f life - the laxity immediately after birth (related to
maternal estrogen) can screw up the measurements.

636
Q

Developmental Dysplasia of the Hip

angles us

A

On ultrasound the alpha angle, should be
more than 60 degrees. Anything less
than that and your cup is not deep enough
to hold your ball. The plain film
equivalent in the acetabular angle, which
is the complimentary angle (and
therefore should be less than 30).

637
Q

Developmental Dysplasia of the Hip

angles, getting them confused

A
Remember that
the “Alpha Angle is the Alpha Male’’ -
and therefore the bigger of the two angles.
**But don’t forget that DDH is more
common in women (not alpha males).
638
Q

Developmental Dysplasia of the Hip

acetabular angle

A
The acetabular angle should decrease
from 30 degrees at birth to 22 degrees at
age 1. DDH is the classic cause o f an
increased angle, but neuromuscular
disorder can also increase it.
639
Q

Developmental Dysplasia of the Hip

position of the femoral epipysis

A
The position of the femoral epiphysis
(or where it will be) should be below
Hilgenreiner’s line “H”, and medial to
Perkin’s Line “P”. Shenton’s Line “S”
should be continuous.
640
Q

Proximal Focal Femoral Deficiency

A

This is a congenital zebra,
which ranges from absent proximal femur to hypoplastic proximal femur. You
get a varus deformity. This is a mimic o f DDH, but DDH will have normal
femur leg length.

641
Q

Slipped Capital Femoral Epiphysis (SCFE)

A

This is a type 1 Salter Harris,
through the proximal femoral physis. What makes this unique is that unlike most SH Is, this
guy has a bad prognosis if not fixed. The classic history is a fat African American
adolescent (age 12-15) with hip pain. It’s bilateral in 1/3 o f cases (both hips don’t usually
present at same time). The frog leg view is the money - this is always the answer on next step questions.

642
Q

Legg-Calve-Perthes

A

This is AVN of the proximal femoral epiphysis. It’s seen more in
boys than girls (4 :1 ), and favors white people around age 5-8. These kids tend to be smaller
than average for their age. This is bilateral about 10% of the time (less than SCFE). The
subchondral lucency (crescent sign) is best seen on a frog leg. Other early signs include an
asymmetric small ossified femoral epiphysis. MRI has more sensitivity. The flat collapsed
femoral head makes it obvious. Sterile joint effusions (transient synovitis) can be associated

643
Q

“Klein’s Line”

A

Drawn along the edge of the
femur and should normally intersect with lateral
superior femoral epiphysis. This line is used to
evaluate for SCFE. When the line doesn’t cross
the lateral epiphysis think SCFE.
***Testable Trivia - Frog Leg View is more
sensitive for this measurement

644
Q

Perthes quick

A

often small white kids

ages 5-8

bilateral 10%

645
Q

SCFE quick

A

overweight black kids

age 12-15

bilateral 30%

646
Q

Septic Arthritis

A

This is serious
business , and considered the most urgent
cause o f painful hip in a child. Wide joint
space (lateral displacement of femoral head),
should prompt an ultrasound, and that should
prompt a joint tap. If you have low suspicion
and don’t want to tap the hip, You could pull
on the leg under fluoro and try and get gas in
the joint. This air arthrogram sign supposedly
excludes a joint effusion (and therefore a
septic joint) - depending on who you ask.

647
Q

Transient Synovitis

A

This is a sterile
(reactive) hip effusion that occurs in the
setting of a systemic illness (usually viral
UR1 or GI). As the name suggests this is
“transient” and goes away in a few days.
This is actually very common. Some
sources will say its the most common hip
disorder in growing children (peak age is
around 5). The ED will be in a full panic and
want you to tap it at 3 am.

648
Q

THIS vs THAT: Transient Synovitis vs Septic Arthritis

overview

A

Telling these apart is actually important for real life (not getting sued) since a septic hip will
fucking destroy the kid’s cartilage (usually if it’s missed for more than 4 days).
Ortho (and in very rare situations a “smart” ED doc)
will use a clinical parameter “the Kocher Criteria ” to
tell them apart.

649
Q

THIS vs THAT: Transient Synovitis vs Septic Arthritis

kocher criteria

A

• Fever
• Inability to walk
• Elevated ESR (or CRP)
. WBC > 12K

650
Q

THIS vs THAT: Transient Synovitis vs Septic Arthritis

bullets

A
• If 3/4 are positive = Septic
• If CRP is negative and the
kid can bear weight it’s
NOT Septic
• CRP is the strongest
independent risk factor for
septic arthritis
651
Q

THIS vs THAT: Transient Synovitis vs Septic Arthritis

typical workup

A
  1. X-Ray Hip series (AP, Lateral, and Frog Leg) which is usually negative in real life but
    will probably show medial joint widening on the exam.
  2. Ultrasound which will show an effusion.
  3. Then a clinical decision based on Kocher Criteria (>2) and “Gut Instinct” to Aspirate
  4. MRI would only be used if/when hip aspiration can’t/hasn’t been performed
652
Q

Rickets

A

Not enough vitamin D. Affects the most
rapidly growing bones (mostly knees and wrists).
Buzzwords “fraying, cupping, and irregularity along
the physeal margin. ” They are at increased risk for
SCFE. “Rachitic rosary” appearance from expansion of
the anterior rib ends at the costochondral junctions. As a
pearl, rickets is never seen in a newborn (Mom’s
vitamin D is still doing its thing).

653
Q

Hypophosphatasia

A

This looks like Rickets in a
newborn. They will have frayed metaphyses and bowed
long bones. The underlying pathology is deficient
serum alkaline phosphatase. There is variability in
severity with lethal perinatal / natal forms, and more mild
adult forms.

654
Q

Scurvy

A

Not enough vitamin C. This is rare as hell outside o f a pirate ship in the 1400s.
For the purpose of trivia (which multiple choice tests love) the following stuff is high yield:
* Does NOT occur before 6 months o f age (maternal stores buffer)
o Bleeding Disorders Common
o Subperiosteal hemorrhage (lifts up the periosteum)
* Hemarthrosis
* “Scorbutic rosary” appearance from expansion of the costochondral junctions (very
similar to rickets).

655
Q

Lead Poisoning

A

This is most commonly seen in kids less than two who eat paint chips.
The classic finding is a wide sclerotic metaphyseal line (lead line), in an area o f rapid
growth (knee). It will not spare the fibula (as a normal variant line might).

656
Q

Lucent Metaphyseal Bands

A
  • This is a classic peds DDx. - LINE.
  • Leukemia
  • Infection (TORCH)
  • Neuroblastoma Mets
  • Endocrine (rickets, Scurvy)
657
Q

Non-Accidental Trauma I NAT

A

Posterior Medial Rib Fracture
Metaphyseal Corner Fractures
Skull Fracture
Solid Organ and Lumen Injury

658
Q

Posterior Medial Rib Fracture

A

In a child under the age o f
3, this is pretty reliable. Supposedly this type o f fracture can
only be made from squeezing a child.

659
Q

Metaphyseal Corner Fractures

A

When this is present in a
non-ambulatory patient (infant) it is HIGHLY specific. The
only exception is obstetric trauma. After age 1, this becomes
less specific.

660
Q

Skull Fracture

A

The general idea is anything other than a parietal bone fracture (which is
supposedly seen more with an actual accident) is concerning.

661
Q

Solid Organ and Lumen Injury

A

Don’t forget about this as a presentation for NAT.
Duodenal hematoma and pancreatitis (from trauma) in an infant - should get you to say
NAT. Just think “belly trauma in a kid that is too young to fall on the handle bars o f their
bike

662
Q

Dating the Fracture:

A

• Periosteal Reaction: This means the fracture is less than a week old.
• Complete healing: This occurs in around 12 weeks.
• Exceptions: Metaphyseal, skull, and costochondral junction fractures will often heal
without any periosteal reaction.

663
Q

C hild A b u s e M im ic s

A

Rickets and 0 1 , can have multiple fractures at different sites and are the two most
commonly described mimics.
Wormian bones and bone mineral density issues are clues that you are dealing with a
mimic. They will have to show you one or the other (or both) if they are gonna get
sneaky.

664
Q

Central Canal

A

Easily identified as an
echogenic line. Although that is sort o f counter
intuitive.

665
Q

Central Canal Why would a structure with fluid in it be

echogenic?

A

The reason is that you are actually
seeing a “central echo complex” - which is the
interface between the anterior median fissure and
the myelinated ventral white matter commissure.
Remember that interfaces between things with
large differences in impedance cause a lot of
reflections (thus an echogenic line).

666
Q

Central Canal

trivia

A

Technically in a newborn the central canal is not even fluid filled (it’s packed with
glial fibrils), but that level of trivia is beyond the scope o f the exam (probably).

667
Q

Low lying cord I Tethered cord

A

Because the canal grows faster than the cord, a
fixed attachment (“tethering”) results in cord stretching and subsequent ischemia. This can be
primary (isolated), or secondary (associated with myelomeningocele, filum terminale lipoma,
or trauma). The secondary types are more likely shown on MR (to showcase the associated
mass - fluid collection), the primary types are more likely shown on US - as a straight
counting game.

668
Q

Low lying cord I Tethered cord

imaging features

A

Low conus (below L2L and thickened filum terminale (> 2mm).

669
Q

Low lying cord I Tethered cord

high yeild trivia

A

A common piece o f trivia used as a distractor is that meningomyelocele is associated with
Chiari malformations, lipomyelomeningocele is NOT.

  • Anal Atresia = High Risk For Occult Cord Problems (including tethering) - should get
    screened
  • Low lying / tethered cords are closely linked with Spina Bifida (tufts o f hair)
  • Low Dimples (below the gluteal crease) Do NOT need screening,
    o These never extend intra-spinally. They might later become a pilonidal sinuses
  • but aren’t ever gonna have shit to do with the cord.
  • High Dimples (above the gluteal crease) DO need screening.
670
Q

Low lying cord I Tethered cord

for screening

A
Low dimples (below the butt crease) don’t get screened, basically everything else does.
Most Likely Question Style: “Which of the following does NOT get screened ?”
(Answer = low dimple).
671
Q
Terminal Ventricle (ventriculus
terminalis):
A

This is a developmental variant.
Normally, a large portion o f the distal cord involutes in a
late stage of spinal cord embryology. Sometimes this
process is not uniform and you get stuck with a stupid
looking cyst at the end o f your cord. These things are
usually small (around 4 mm), and cause no symptoms.
Sometimes they can get very big (like this example) and
cause some neurologic symptoms.

672
Q

Pars Interarticularis Defects (Spondylolysis

A

This is considered a fatigue or stress fracture, probably developing in
childhood. It is a classic cause o f back pain in an adolescent athlete.
Although they are usually not symptomatic (only 25% are). The
process represents a hole / break in the connecting bone between the
superior and inferior articular facets. If there is forward “slippage”
you can deploy the word spondylolisthesis.
Almost always (90% +) you see this at L5 (2nd most common at L4).
They tend to have more spondylolisthesis and associated degenerative
change at L4-L5 than L5-S1. They can be seen on the oblique plain
film as a “collar on the scottie dog.” The collar on the “scotty dog”
appearance on an oblique plain film is probably the most common
way they show this in case books and conferences. On the AP view
this can be a cause o f a sclerotic pedicle (the contralateral pedicle -
from wiggle stress). On CT it is usually more obvious with the break
clearly demonstrated.
**Pars Defects with anterolisthesis will have neuroforaminal stenosis,
with spinal canal widening (when severe will have spinal canal
stenosis as well). If the process is purely a degenerative
spondylolisthesis (not much slippage), the resulting facet arthropathy
will favor the canal with less severe effects on the neuro foramina

673
Q

Spinal Dysraphism

A

You can group these as open or closed (closed with and without a mass). Open means neural
tissue exposed through a defect in bone and skin (spina bifida aperta). Closed means the
defect is covered by skin (spina bifida occulta).

674
Q

Open Spinal Dysraphisms

A

This is the result o f a failure o f the closure of the primary
neural tube, with obvious exposure o f the neural placode through a midline defect o f the skin.
You have a dorsal defect in the posterior elements. The cord is going to be tethered. There
is an association with diastematomyelia and Chiari II malformations. Early surgery is the
treatment / standard of care.

675
Q

Open Spinal Dysraphisms

Myelocele

A

This is the more rare type where the neural

placode is flush with the skin.

676
Q

Open Spinal Dysraphisms

Myelomeningocele

A

This is the more common type
(98%) where the neural placode protrudes above the
skin. These are more common with Chiari II
malformations.

677
Q

Closed Spinal Dysraphisms with Subcutaneous Mass

Meningocele

A

This is herniation o f a CSF filled sac through a
defect in the posterior elements (spina bifida). It is most typical
in the lumbar or sacral regions. Although they can occur in the
cervical spine. They may be anterior (usually pre-sarcral). An
important point is that neural tissue is NOT present in the
sac.

678
Q

Closed Spinal Dysraphisms with Subcutaneous Mass

Lipomyelocele / Lipomyelomeningocele

A

These are lipomas with a dural defect. On
exam you are going to have a subcutaneous fatty mass above the gluteal
crease. These are 100% associated with tethered cord
(myelomeningocele may or may not).

679
Q

Closed Spinal Dysraphisms with Subcutaneous Mass

Terminal Myelocystocele

A

This is a herniation o f a terminal

syrinx into a posterior meningocele via a posterior spinal defect

680
Q

Closed Spinal Dysraphisms without Subcutaneous Mass

Intradural lipomas

A

Most common in the thoracic spine along the dorsal aspect.

They don’t need to be (but can be) associated with posterior element defects.

681
Q

Closed Spinal Dysraphisms without Subcutaneous Mass

Fihrolipoma o f thefilum terminate

A

This is often an incidental finding “fatty filum”.
There will be a linear T1 bright structure in the filum terminale. The filum is not
going to be unusually thickened and the conus will be normally located.

682
Q

Closed Spinal Dysraphisms without Subcutaneous Mass

Tight filum terminale

A

This is a thickened filum terminale (> 2 mm), with a low lying
conus (below the inferior endplate of L2). You may have an associated terminal lipoma.
The “tethered cord syndrome” is based on the clinical findings o f low back pain and leg
pain plus urinary bladder dysfunction. This is the result o f stretching the cord with
growth o f the canal.

683
Q

Closed Spinal Dysraphisms without Subcutaneous Mass

Dermal Sinus

A

This is an epithelial lined tract that extends from the skin to deep soft
tissues (sometimes the spinal canal, sometimes a dermoid or lipoma). These are T1 low
signal (relative to the background high signal from fat).

684
Q

Diastematomyelia

A

This describes a sagittal split in the spinal cord. They almost always
occur between T9-S1, with normal cord both above and below the split. You can have two thecal
sacs (or just one), and each hemi-cord has its own central canal and dorsal/ventral horns.
Classification systems are based on the presence / absence o f an osseous or fibrous spur and
duplication or non-duplication o f the thecal sac.

685
Q

Caudal Regression

A

This is a spectrum o f defects in the caudal region that ranges from
partial agenesis o f the coccyx to lumbosacral agenesis. The associations to know are VACTERL
and Currarino triad. Think about this with maternal diabetes. “Blunted sharp” high terminating
cord is classic, with a “shield sign” from the opposed iliac bones (no sacrum).

686
Q

Caudal Regression

trickery

A

Note than the Meningocele of
Currarino is Anterior. It’s not posterior. A
potential deployment o f fuckery is to put
“Posterior” Sacral Meningocele as a
distractor for Currarino Triad.

687
Q

Currarino Triad

A
Anterior Sacral Meningocele,
• •
•• •
• Anorectal malformation,
Sacrococcygeal osseous defect
(scimitar sacrum).